You are on page 1of 82

AE06/AC04/AT04 SIGNALS & SYSTEMS

TYPICAL QUESTIONS & ANSWERS


PART– I
OBJECTIVE TYPE QUESTIONS

Each Question carries 2 marks.


Choose the correct or best alternative in the following:
n
Q.1 The discrete-time signal x (n) = (-1) is periodic with fundamental period
(A) 6 (B) 4
(C) 2 (D) 0

Ans: C Period = 2

Q.2 The frequency of a continuous time signal x (t) changes on transformation


from x (t) to x ( α t), α > 0 by a factor
1
(A) α . (B) .
α
(C) α .2
(D) α.
Transform
Ans: A x(t) x(αt), α > 0
α>1 compression in t, expansion in f by α.
α<1 expansion in t, compression in f by α.

Q.3 A useful property of the unit impulse δ (t) is that


(A) δ (at) = a δ (t) . (B) δ (at) = δ (t) .
1
(C) δ (at) = δ (t) . (D) δ(at ) = [δ(t )]a .
a
Ans: C Time-scaling property of δ(t):
δ(at) = 1 δ(t), a > 0
a

Q.4 The continuous time version of the unit impulse δ (t) is defined by the pair of relations

1
AE06/AC04/AT04 SIGNALS & SYSTEMS

t=0 ∞
(A) δ (t) = 
1
(B) δ (t) = 1, t = 0 and ∫ δ (t) dt = 1 .
0 t ≠0 . -∞
∞ 1, t ≥ 0
(C) δ (t) = 0, t ≠ 0 and ∫ δ (t) dt = 1 . (D) δ(t ) =  .
-∞ 0, t < 0

Ans: C δ(t) = 0, t ≠ 0 → δ(t) ≠ 0 at origin


+∞
∫ δ(t) dt = 1 → Total area under the curve is unity.
-∞
[δ(t) is also called Dirac-delta function]

Q.5 Two sequences x1 (n) and x2 (n) are related by x2 (n) = x1 (- n). In the z- domain, their
ROC’s are
(A) the same. (B) reciprocal of each other.
(C) negative of each other. (D) complements of each other.
. z
Ans: B x1(n) X1(z), RoC Rx
z Reciprocals
x2(n) = x1(-n) X1(1/z), RoC 1/ Rx

Q.6 The Fourier transform of the exponential signal e jω 0 t is


(A) a constant. (B) a rectangular gate.
(C) an impulse. (D) a series of impulses.

Ans: C Since the signal contains only a high frequency ωo its FT must be an impulse at
ω = ωo

ω
Q.7 If the Laplace transform of f (t ) is , then the value of Lim f (t )
(s 2
+ ω2 ) t →∞
(A) cannot be determined. (B) is zero.
(C) is unity. (D) is infinity.

L
Ans: B f(t) ω
s2 + ω2

Lim f(t) = Lim s F(s) [Final value theorem]


t ∞ s 0

= Lim sω =0
s 0 s2 + ω2

Q.8 The unit impulse response of a linear time invariant system is the unit step
function u (t ) . For t > 0, the response of the system to an excitation
e −at u (t ), a > 0, will be
1 − e −at
(A) ae −at . (B) .
a

2
AE06/AC04/AT04 SIGNALS & SYSTEMS

(C) a (1 − e − at ) . (D) 1 − e − at .

Ans: B
h(t) = u(t); x(t) = e-at u(t), a > 0
System response y(t) = L−1  .  1 1

s s + a

= L−1  −
1 1 1 
a  s s + a 
= 1 (1 - e-at)
a
0
Q.9 The z-transform of the function ∑ δ(n − k ) has the following region of convergence
k = −∞
(A) z > 1 (B) z = 1
(C) z < 1 (D) 0 < z < 1
0
Ans: C x(n) = ∑ δ(n-k)
k = -∞
0
x(z) = ∑ z-k = …..+ z3 + z2 + z + 1 (Sum of infinite geometric series)
k = -∞
= 1 , z < 1
1–z

Q.10 The auto-correlation function of a rectangular pulse of duration T is


(A) a rectangular pulse of duration T.
(B) a rectangular pulse of duration 2T.
(C) a triangular pulse of duration T.
(D) a triangular pulse of duration 2T.
Ans: D
T/2
RXX (τ) = 1 ∫ x(τ) x(t + τ) dτ triangular function of duration 2T.
T -T/2

Q.11 The Fourier transform (FT) of a function x (t) is X (f). The FT of dx (t ) / dt will be
(A) dX(f ) / df . (B) j2πf X(f ) .
(C) jf X (f ) . (D) X (f ) / ( jf ) .

Ans: B (t) = 1 ∫ X(f) ejωt dω
2π - ∞

d x = 1 ∫ jω X(f) ejωt dω
dt 2π - ∞
∴ d x ↔ j 2π f X(f)
dt

Q.12 The FT of a rectangular pulse existing between t = − T / 2 to t = T / 2 is a


(A) sinc squared function. (B) sinc function.
(C) sine squared function. (D) sine function.
3
AE06/AC04/AT04 SIGNALS & SYSTEMS

Ans: B x(t) = 1, -T ≤ t ≤ T
2 2
0, otherwise

+∞ +T/2 +T/2
X(jω) = ∫ x(t) e-jωt dt = ∫ e-jωt dt = e-jωt
-∞ -T/2 jω
-T/2

= - 1 (e-jωT/2 - ejωT/2) = 2 ejωT/2 - e-jωT/2


jω ω 2j

= 2 sin ωT = sin(ωT/2) .T
ω 2 ωT/2

Hence X(jω) is expressed in terms of a sinc function.

Q.13 An analog signal has the spectrum shown in Fig. The minimum sampling
rate needed to completely represent this signal is
(A) 3 KHz .
(B) 2 KHz .
(C) 1 KHz .
(D) 0.5 KHz .

Ans: C For a band pass signal, the minimum sampling rate is twice the
bandwidth, which is 0.5 kHz here.

Q.14 A given system is characterized by the differential equation:


d 2 y(t ) dy(t )
− − 2 y(t ) = x (t ) . The system is:
dt 2 dt
(A) linear and unstable. (B) linear and stable.
(C) nonlinear and unstable. (D) nonlinear and stable.

Ans:A d2y(t) – dy(t) – 2y(t) = x(t), x(t) x(t)


h(t) y(t)
dt2 dt system
The system is linear . Taking LT with zero initial conditions, we get
s2Y(s) – sY(s) – 2Y(s) = X(s)

or, H(s) = Y(s) = 1 = 1


2
X(s) s – s – 2 (s –2)(s + 1)

Because of the pole at s = +2, the system is unstable.

Q.15 The system characterized by the equation y(t ) = ax (t ) + b is


(A) linear for any value of b. (B) linear if b > 0.
(C) linear if b < 0. (D) non-linear.

4
AE06/AC04/AT04 SIGNALS & SYSTEMS

Ans: D The system is non-linear because x(t) = 0 does not lead to y (t) = 0, which
is a violation of the principle of homogeneity.

Q.16 Inverse Fourier transform of u (ω) is


1 1 1
(A) δ(t ) + . (B) δ(t ) .
2 πt 2
1
(C) 2δ(t ) + . (D) δ(t ) + sgn (t ) .
πt

FT
Ans: A x(t) = u(t) X(jω) = π δ(ω) + 1

Duality property: X(jt) 2π x(-ω)

u(ω) 1 δ(t) + 1
2 πt

Q.17 The impulse response of a system is h (n ) = a n u (n ) . The condition for the system to
be BIBO stable is
(A) a is real and positive. (B) a is real and negative.
(C) a > 1 . (D) a < 1 .
+∞ +∞
Ans: D Sum S = ∑ h(n) = ∑  an u(n) 
n = -∞ n = -∞
+∞
≤ ∑ a n ( u(n) = 1 for n ≥ 0 )
n=0
≤ 1 if a < 1.
1- a

Q.18 If R1 is the region of convergence of x (n) and R 2 is the region of convergence of


y(n), then the region of convergence of x (n) convoluted y (n) is
(A) R1+ R 2 . (B) R1−R 2 .
(C) R 1 ∩ R 2 . (D) R1∪R 2 .
z
Ans:C x(n) X(z), RoC R1
z
y(n) Y(z), RoC R2
z
x(n) * y(n) X(z).Y(z), RoC at least R1 ∩ R2

Q.19 The continuous time system described by y(t ) = x t 2 is ( )


(A) causal, linear and time varying.
(B) causal, non-linear and time varying.
(C) non causal, non-linear and time-invariant.
(D) non causal, linear and time-invariant.

5
AE06/AC04/AT04 SIGNALS & SYSTEMS

Ans: D
y(t) = x(t2)
y(t) depends on x(t2) i.e., future values of input if t > 1.
System is anticipative or non-causal
α x1(t) → y1(t) = α x1(t2)
β x2(t) → y2(t) = β x2(t2)
α x1(t) + β x2(t) → y(t) = αx1(t2) +β x2(t2) = y1(t) + y2(t)
System is Linear
System is time varying. Check with x(t) = u(t) – u(t-z) → y(t) and
x1(t) = x(t – 1) → y1(t) and find that y1(t) ≠ y (t –1).

Q.20 If G(f) represents the Fourier Transform of a signal g (t) which is real and odd
symmetric in time, then G (f) is
(A) complex. (B) imaginary.
(C) real. (D) real and non-negative.

FT
Ans: B g(t) G(f)

g(t) real, odd symmetric in time

G*(jω) = - G(jω); G(jω) purely imaginary.

Q.21 For a random variable x having the PDF shown in the Fig., the mean and the
variance are, respectively,
(A) 1 and 2 .
2 3
(B) 1 and 4 .
3
(C) 1 and 2 .
3
(D) 2 and 4 .
3

+∞
Ans:B Mean = µx(t) = ∫ x fx(t) (x) dx
-∞
3
= ∫ x 1 dx = 1 x2 3 = 9 – 1 1 = 1
-1 4 4 2 -1 2 2 4

+∞
Variance = ∫ (x - µx)2 fx (x) dx
-∞

3
= ∫ (x - 1)2 1 d(x-1)
-1 4
6
AE06/AC04/AT04 SIGNALS & SYSTEMS

= 1 (x - 1)3 3 = 1 [8 + 8] = 4
4 3 -1 12 3

Q.22 If white noise is input to an RC integrator the ACF at the output is proportional to
− τ   −τ 
(A) exp .

(B) exp .
 RC   RC 
(C) exp( τ RC ) . (D) exp(- τ RC ) .

Ans: A

RN(τ) = N0 exp -  τ 
4RC RC

x (n ) = a , a < 1 is
n
Q.23
(A) an energy signal.
(B) a power signal.
(C) neither an energy nor a power signal.
(D) an energy as well as a power signal.

Ans: A +∞ ∞ ∞ ∞
Energy = ∑ x2(n) = ∑ a2 = ∑ (a2) = 1+ 2 ∑ a2
n=-∞ n=-∞ n=-∞ n=1

= finite since a < 1


∴This is an energy signal.

Q.24 The spectrum of x (n) extends from − ω o to + ωo , while that of h(n) extends

from − 2ω o to + 2ω o . The spectrum of y(n ) = ∑ h(k ) x(n − k ) extends
k = −∞
from
(A) − 4ω o to + 4ω o . (B) − 3ω o to + 3ω o .
(C) − 2ω o to + 2ω o . (D) − ω o to + ω o
.
Ans: D Spectrum depends on H( ejω) X( ejω) Smaller of the two ranges.

Q.25 The signals x1 (t ) and x 2 (t ) are both bandlimited to (− ω1 , + ω1 ) and


(− ω 2 , + ω 2 ) respectively. The Nyquist sampling rate for the signal x1 (t ) x 2 (t )
will be
(A) 2ω1 if ω1 > ω 2 . (B) 2ω 2 if ω1 < ω 2 .
(C) 2 (ω1 + ω 2 ) . (D) (ω1 + ω 2 ) .
2

Ans: C Nyquist sampling rate = 2(Bandwidth) = 2(ω1 – (-ω2)) = 2(ω1 + ω2)

Q.26 (
If a periodic function f(t) of period T satisfies f (t ) = −f t + T
2
), then in its Fourier
series expansion,
7
AE06/AC04/AT04 SIGNALS & SYSTEMS

(A) the constant term will be zero.


(B) there will be no cosine terms.
(C) there will be no sine terms.
(D) there will be no even harmonics.

Ans:
T T/2 T T/2 T/2
1 ∫ f(t) dt = 1 ∫ f(t) dt + ∫f(t) dt = 1 ∫ f(t) dt + ∫ f(τ + T/2)dτ = 0
T 0 T 0 T/2 T 0 0

Q.27 A band pass signal extends from 1 KHz to 2 KHz. The minimum sampling frequency
needed to retain all information in the sampled signal is
(A) 1 KHz. (B) 2 KHz.
(C) 3 KHz. (D) 4 KHz.

Ans: B
Minimum sampling frequency = 2(Bandwidth) = 2(1) = 2 kHz

Q.28 The region of convergence of the z-transform of the signal 2 n u (n ) − 3n u (− n − 1)


(A) is z > 1 . (B) is z < 1 .
(C) is 2 < z < 3 . (D) does not exist.
Ans:

2nu(n) 1 , z > 2


1 –2 z -1

3n u(-n-1) 1 , z < 3


1 – 3z -1

ROC is 2 < z < 3.


(e −2 − 2)z is
(z − e −2 )(z − 2)
Q.29 The number of possible regions of convergence of the function

(A) 1. (B) 2.
(C) 3. (D) 4.

Ans: C
Possible ROC’s are z > e-2 , z < 2 and e-2 < z < 2

Q.30 The Laplace transform of u(t) is A(s) and the Fourier transform of u(t) is B( jω) .
Then
1 1
(A) B( jω) = A (s ) s = jω . (B) A(s ) = but B( jω) ≠ .
s jω
1 1 1 1
(C) A(s ) ≠ but B( jω) = . (D) A(s ) ≠ but B( jω) ≠ .
s jω s jω
L
Ans: B u(t) A(s) = 1
s
8
AE06/AC04/AT04 SIGNALS & SYSTEMS

F.T
u(t) B(jω) = 1 + π δ(ω)

A(s) = 1 but B(jω) ≠ 1


s jω

Q.31 Given a unit step function u(t), its time-derivative is:


(A) a unit impulse. (B) another step function.
(C) a unit ramp function. (D) a sine function.

Ans: A

Q.32 The impulse response of a system described by the differential equation


d2y
+ y( t ) = x ( t ) will be
dt 2
(A) a constant. (B) an impulse function..
(C) a sinusoid. (D) an exponentially decaying function.

Ans: C
sin(πu )
Q.33 The function is denoted by:
(πu )
(A) sin c(πu). (B) sin c(u).
(C) signum. (D) none of these.

Ans: C

Q.34 The frequency response of a system with h(n) = δ(n) - δ(n-1) is given by
(A) δ(ω) - δ(ω - 1). (B) 1 - ejω.
(C) u(ω) – u(ω -1). (D) 1 – e-jω.

Ans: D

Q.35 The order of a linear constant-coefficient differential equation representing a system


refers to the number of
(A) active devices. (B) elements including sources.
(C) passive devices. (D) none of those.

Ans: D

Q.36 z-transform converts convolution of time-signals to


(A) addition. (B) subtraction.
(C) multiplication. (D) division.

Ans: C

Q.37 Region of convergence of a causal LTI system


(A) is the entire s-plane. (B) is the right-half of s-plane.
(C) is the left-half of s-plane. (D) does not exist.

9
AE06/AC04/AT04 SIGNALS & SYSTEMS

Ans: B

Q.38 The DFT of a signal x(n) of length N is X(k). When X(k) is given and x(n) is computed
from it, the length of x(n)
(A) is increased to infinity (B) remains N
(C) becomes 2N – 1 (D) becomes N2

Ans: A

Q.39 The Fourier transform of u(t) is


1
(A) . (B) j2πf.
j2πf
1
(C) . (D) none of these.
1 + j2πf

Ans: D

Q.40 For the probability density function of a random variable X given by


f x ( x ) = 5e − Kx u ( x ) , where u(x) is the unit step function, the value of K is
1 1
(A) (B)
5 25
(C) 25 (D) 5

Ans: D

Q.41 The system having input x(n) related to output y(n) as y(n) = log10 x (n ) is:
(A) nonlinear, causal, stable. (B) linear, noncausal, stable.
(C) nonlinear, causal, not stable. (D) linear, noncausal, not stable.

Ans: A

Q.42 To obtain x(4 – 2n) from the given signal x(n), the following precedence (or priority)
rule is used for operations on the independent variable n:
(A) Time scaling → Time shifting → Reflection.
(B) Reflection → Time scaling → Time shifting.
(C) Time scaling → Reflection → Time shifting.
(D) Time shifting → Time scaling → Reflection.

Ans: D

Q.43 The unit step-response of a system with impulse response h(n) = δ(n) – δ(n – 1) is:
(A) δ(n – 1). (B) δ(n).
(C) u(n – 1). (D) u(n).

Ans: B

10
AE06/AC04/AT04 SIGNALS & SYSTEMS

Q.44 If φ(ω) is the phase-response of a communication channel and ωc is the channel


dφ(ω)
frequency, then − ω = ωc represents:

(A) Phase delay (B) Carrier delay
(C) Group delay (D) None of these.

Ans: C

Q.45 Zero-order hold used in practical reconstruction of continuous-time signals is


mathematically represented as a weighted-sum of rectangular pulses shifted by:
(A) Any multiples of the sampling interval.
(B) Integer multiples of the sampling interval.
(C) One sampling interval.
(D) 1 second intervals.

Ans: B

ℑ  dx ( t ) 
Q.46 If x (t ) ↔ X (s), then ℑ  is given by:
 dt 
dX(s) X (s) x −1 (0)
(A) . (B) − .
ds s s
(C) sX (s) − x (0 − ) . (D) sX(s) – sX(0).

Ans: C

Q.47 The region of convergence of the z-transform of the signal x(n) ={2, 1, 1, 2}

is n=0
(A) all z, except z = 0 and z = ∞ (B) all z, except z = 0.
(C) all z, except z = ∞. (D) all z.

Ans: A

Q.48 When two honest coins are simultaneously tossed, the probability of two heads on any
given trial is:
3
(A) 1 (B)
4
1 1
(C) (D)
2 4

Ans: D

Q.49 Let u[n] be a unit step sequence. The sequence u[ N − n] can be described as
1, n<N 1, n≤N
(A) x[n] =  (B) x[n] = 
0, otherwise 0, otherwise
1, n>N 1, n≥N
(C) x[n] =  (D) x[n] = 
0, otherwise 0, otherwise
11
AE06/AC04/AT04 SIGNALS & SYSTEMS

1, n ≤ N
Ans (B) x[n] = 
0, otherwise
Here the function u(-n) is delayed by N units.
Q.50 A continuous-time periodic signal x(t ) , having a period T, is convolved with
itself. The resulting signal is
(A) not periodic (B) periodic having a period T
(C) periodic having a period 2T (D) periodic having a period T/2

Ans (B) periodic having a period T


Convolution of a periodic signal (period T) with itself will give the same period
T.
Q.51 If the Fourier series coefficients of a signal are periodic then the signal must be
(A) continuous-time, periodic (B) discrete-time, periodic
(C) continuous-time, non-periodic (D) discrete-time, non-periodic

Ans B) discrete-time, periodic


This is the property of the discrete-time periodic signal.
Q.52 The Fourier transform of a signal x(t ) = e 2t u (−t ) is given by
1 2
(A) . (B)
2 − jω 1 − jω
1 2
(C) (D)
j2 − ω j2 − ω
1
Ans (A)
2 − jω
1 1
FT u(t) = . Therefore, FT of u(-t) = . If a function x(t) is multiplied
jω − jω
by e 2t , then its FT will be F ( jω ) jω → jω − 2 . Hence the answer.

1
For the function H ( jω ) = , maximum value of group delay is
Q.53 2 + 2 jω + ( jω )
2

(A) 1 (B) 1/2


(C) 2 (D) 3

Ans None of the given answers is correct.


Q.54 A continuous-time signal x(t ) is sampled using an impulse train. If X ( jω ) is the
Fourier transform of x(t ) , the spectrum of the sampled signal can be expressed as
∞ ∞
(A) ∑ X ( jω + kω )δ (ω )
k = −∞
s (B) ∑ X ( jkω ) * δ (ω + kω )
k = −∞
s

∞ ∞
(C) ∑ X ( jω ) * δ (ω + kω )
k = −∞
s (D) ∑ X ( jω )δ (ω + kω )
k = −∞
s

12
AE06/AC04/AT04 SIGNALS & SYSTEMS


Ans (A) ∑ X ( jω + kω )δ (ω )
k = −∞
s

Since the spectrum consists of various harmonics k = −∞ to ∞ and discretely


spread at an interval of fundamental frequency fs. Hence the answer.
Q.55 The region of convergence of a causal finite duration discrete-time signal is
(A) the entire z-plane except z = 0
(B) the entire z-plane except z = ∞
(C) the entire z-plane
(D) a strip in z-plane enclosing jω -axis

Ans (A) The entire z-plane except z = 0


n2
X ( z) = ∑ x[n]z
n = n1
−n
. This sum should converge provided each term in the sum is

finite. However, if there is a non-zero causal component for n2>0, then X(z) will
have a term involving z-1 and thus ROC cannot include z = 0.
Q.56 ( )
Let H e jω be the frequency response of a discrete-time LTI system, and
H I (e jω ) be the frequency response of its inverse. Then,

(A) H (e jω )H I (e jω ) = 1 (B) H (e jω )H I (e jω ) = δ (ω )
(C) H (e jω ) * H I (e jω ) = 1 ( ) ( )
(D) H e jω * H I e jω = δ (ω )

( ) ( )
Ans (A) H e jω H I e jω = 1
Since H (e jω ) and H I (e jω ) are the inverse of each other, their product should
equal 1.
1 1
Q.57 The transfer function of a stable system is H ( z ) = −1
+ .
1 − 0. 5 z 1 − 2 z −1
Its impulse response will be
(A) (0.5) u[n] + (2 ) u[n] (B) − (0.5) u[− n − 1] + (2 ) u[n]
n n n n

(C) (0.5)n u[n] − (2)n u[−n − 1] (D) − (0.5) u[− n − 1] − (2 ) u[−n − 1]


n n

Ans (C) (0.5) u[n] − (2 ) u[− n − 1]


n n

(A) and (C) are the possible IFTs of the given system function. However,
the system is stable; therefore (C) is the only correct answer.
Q.58 The probability cumulative distribution function must be monotone and
(A) increasing (B) decreasing
(C) non-increasing (D) non-decreasing

Ans (D) non-decreasing


The probability cumulative distribution function increases to 1 monotonically and
there after remains constant.

Q.59 The average power of the following signal is

13
AE06/AC04/AT04 SIGNALS & SYSTEMS

A2
(A) (B) A 2
2
(C) AT12 (D) A 2T1

Ans: (D)
T1 / 2
W= ∫−T1 / 2
x(t ) 2 dt = A 2 T1

Q.60 Convolution is used to find:


(A) The impulse response of an LTI System
(B) Frequency response of a System
(C) The time response of a LTI system
(D) The phase response of a LTI system

Ans: (C)
k =∞
Time response y ( n ) = x ( n ) * h( n ) = ∑ x(k )h(n − k )
k = −∞

Q.61 The Fourier Transform of a rectangular pulse is


(A) Another rectangular pulse (B) Triangular pulse
(C) Sinc function (D) Impulse.

Ans: (C)
This can be seen by putting the value of pulse function in the definition of Fourier
transform.
Q.62 The property of Fourier Transform which states that the compression in time
domain is equivalent to expansion in the frequency domain is
(A) Duality. (B) Scaling.
(C) Time Scaling. (D) Frequency Shifting.

Ans: (B)
Substituting the square pulse function f(t) in

F ( jω ) = ∫
−∞
f (t )e jωt dt
gives the sinc function.
Q.63 What is the Nyquist Frequency for the signal
x(t) =3 cos 50πt +10 sin 300πt – cos100πt ?
(A) 50 Hz (B) 100 Hz
(C) 200 Hz (D) 300 Hz

14
AE06/AC04/AT04 SIGNALS & SYSTEMS

Ans: (D) Here the highest frequency present in the signal is ω m = 300π or
f m = 150 Hz. Therefore the Nyquist frequency f s = 2 f m = 300 Hz.

Q.64 The step response of a LTI system when the impulse response h(n) is unit step
u(n) is
(A) n+1 (B) n
(C) n-1 (D) n 2

Ans: (A)
∞ 6
y ( n ) = x ( n ) * h ( n) = u ( n) * u ( n ) = ∑
k = −∞
u ( k )u ( n − k ) = ∑ u (k )u(n − k )
k =0
y (0) = 1, y (1) = 2, y (2) = 3, ...., y (n) = (n + 1)
y (n) = (n + 1) .

Q.65 The Laplace transform of u(t) is


1
(A) (B) s2
s
1
(C) (D) s
s2

Ans: (A)

Substituting f (t ) = u (t ) in the relation F ( s ) = ∫ f (t )e − st dt gives the answer.
o

The function which has its Fourier transform, Laplace transform, and Z transform
Q.66
unity is
(A) Gausian (B) impulse
(C) Sinc (D) pulse
Ans: (B)
Substituting f(t) = δ (t ) in the definitions of Fourier, Laplace and Z-transform, we
get the transforms in each case as 1.
Q.67 The Z transform of δ (n − m ) is
(A) z − n (B) z − m
1 1
(C) (D)
z−n z−m

Ans: (B)
The Z-transform of a delayed function f(n-m) is z-m times the Z-transform of the
function f(n).
1
Q.68 If the joint probability pdf of f (x, y ) = , 0 ≤ x, y ≤ 2, P(x + y ≤ 1) is
4
1 1
(A) (B)
8 16
1 1
(C) (D)
4 2

15
AE06/AC04/AT04 SIGNALS & SYSTEMS

Ans: (A)
1 1− y 1 1 1 1− y 1 1 1
P( x + y ) = ∫∫
0 0 4
dxdy =
4 ∫ x0
0
dy =
4 ∫ (1 − y )dy = 8 .
0

Q.69 The period of the signal x(t ) = 10 sin 12πt + 4 cos 18πt is
π 1
(A) (B)
4 6
1 1
(C) (D)
9 3
Ans: (D)
There are two waveforms of frequencies 6 and 9, respectively. Hence the
combined frequency is the highest common factor between 6 and 9,i.e., 3. Hence
period is 1/3.

Q.70 The autocorrelation of a rectangular pulse is


(A) another rectangle pulse (B) Square pulse
(C) Triangular pulse (D) Sinc pulse
Ans: (C)
Autocorrelation involves the integration of a constant which gives a ramp
function. Hence the triangular pulse.

Q.71 If the Fourier series coefficients of a signal are periodic then the signal must be
(A) continuous-time, periodic (B) discrete-time, periodic
(C) continuous-time, non periodic (D) discrete-time, non perodic
Ans: (B)
It is the property of the discrete-time periodic signal.

Q.72 The area under the curve ∫ δ (t ) dt is


−∞
(A) ∞ (B) unity
(C) 0 (D) undefined
Ans: (B)

∫ δ (t ) = 1
By definition of delta function, −∞

Q.73 A transmission is said to be _____________ if the response of the system is exact


replica of the input signal.
(A) LTI (B) Distorted
(C) Distortionless (D) Causal
Ans: (C)
Since y (n) = x(n) .

16
AE06/AC04/AT04 SIGNALS & SYSTEMS

Q.74 Laplace Transform of tn is always equal to


n n!
(A) n (B) n
s s
n!
(C) n +1 (D) All
s
Ans: (C)
∞ n!
£
tn = ∫
0
t n e − st dt =
s n +1

Q.75 For a stable system


(A) z <1 (B) z =1
(C) z >1 (D) z ≠1
Ans: (A)
For the system to be stable, the ROC should include the unit circle.

Q.76 The region of convergence of a causal finite duration discrete time signal is
(A) The entire ‘z’ plane except z = 0
(B) The entire ‘z’ plane except z = ∞
(C) The entire ‘z’ plane
(D) A strip in z-plane
Ans: (A)
The ROC of the causal finite duration will have negative power of z. The ROC is
the entire z-plane except z = 0.

Q.77 The CDF for a certain random variable is given as

0, −∞< x≤0


 2
F X ( x ) = kx , 0 < x ≤ 10
100k , 10 < x < ∞

The value of k is
(A) 100 (B) 50
(C) 1/50 (D) 1/100

Ans: (D)
From the given F(x), we get
dF ( x )
= 0 + 2kx + 0 = 2kx
dx
10

∴ 2kxdx = 1
0

or 100k = 1 → k = 1 / 100

Q.78 The group delay function τ(ω) is related to phase function φ(ω) as
−d d
(A) τ (ω ) = φ (ω ) (B) τ (ω ) = φ (ω )
dω dω 2
17
AE06/AC04/AT04 SIGNALS & SYSTEMS

d2 d2
(C) τ (ω ) = φ (ω ) (D) τ (ω ) = φ (ω )
dω 2 dω
Ans: (A): By definition.

Q.79 Two sequences x1 (n ) and x 2 (n ) are related by x 2 (n ) = x1(− n ) . In the


Z-domain, their ROCs are
(A) same (B) reciprocal of each other
(C) negative of each other (D) complement of each other
Ans: (B)
ROC of Z [x2 (n)] is outside the circle of radius r2 while ROC of Z [x1 (−n)] is
inside the circle of radius r1 such that r2 = 1/ r1.

Q.80 The autocorrelation of a sinusoid is


(A) Sinc pulse (B) another sinusoid
(C) Rectangular pulse (D) Triangular pulse
Ans: (B)

φ XX (t ) = ∫ x(τ ) x(τ − t )dτ
−∞

= ∫−∞
A sin ωτ × A sin ω (τ − t )dτ

A2 ∞
=
2 −∞ ∫
[(cos t − cos 2ωτ . cos t − sin 2ωτ . sin t ]dτ

A2  π
= K [(cos t − cos 2ωτ . cos t − sin 2ωτ . sin t ]dτ 

2  −π 
2
K  [cos t ]dτ  = K ' cos t
A π
=
2  −π ∫  .
Thus the autocorrelation is a sinusoid.

Q.81 Which of the following is true for the system represented by y (n ) = x(− n )
(A) Linear (B) Time invariant
(C) Causal (D) Non Linear
Ans.: (A)
The given function is of the form y = mx . Hence linear.

Q.82 The Fourier transform of impulse function is


(A) δ(ω) (B) 2πω
(C) 1 (D) sinc f

Ans: (C)

δ (t ) = ∫ δ (t )e − jωt dt = 1
FT of −∞

Q.83 Convolution is used to find


(A) amount of similarity between the signals
18
AE06/AC04/AT04 SIGNALS & SYSTEMS

(B) response of the system


(C) multiplication of the signals
(D) Fourier transform
Ans: (B)
Convolution of the input signal x(n) and the impulse response h(n) is given by

y ( n ) = x ( n) * h( n ) = ∑ x ( k ) h(n − k )
k = −∞ , where y (n) is the response of the system.

Q.84 The final value of x(t ) = 2 + e − 3t u (t ) is[ ]


(A) 2 (B) 3
(C) e − 3t (D) 0
Ans: (A)
−3t
[
Final value = Lt t →∞ x(t ) = Lt t →∞ 2 + e u (t ) = 2 . ]
Q.85 Discrete time system is stable if the poles are
(A) within unit circle (B) outside unit circle
(C) on the unit circle (D) None
Ans: (A)
The ROC should include the unit circle.

Q.86 The z transform of − u (− n − 1) is


1 z
(A) (B)
1− z 1− z
1 z
(C) (D)
1 − z −1 1 − z −1
Ans: (C)
−∞
z 1
z[− u (−n − 1)] = − ∑ [u (−n − 1)]z − n = −[ z + z 2 + z 3 + ...] = =
n = −1 z − 1 1 − z −1

−π t2
Q.87 The area under Gaussian pulse ∫e dt is
−∞
(A) Unity (B) Infinity
(C) Pulse (D) Zero

Ans: (A)
2
e − πt dt =
∞ ∞ ∞
e − x 2π x e − x dx =1.
x

−∞ ∫
−∞ π
dx = 2 π ∫
−∞

Q.88 The spectral density of white noise is


(A) Exponential (B) Uniform
(C) Poisson (D) Gaussian

19
AE06/AC04/AT04 SIGNALS & SYSTEMS

Ans: (B)
The distribution of White noise is homogeneous over all frequencies. Power
spectrum is the Fourier transform of the autocorrelation function. Therefore,
power spectral density of white noise is uniform.

20
AE06/AC04/AT04 SIGNALS & SYSTEMS

PART – II
NUMERICALS & DERIVATIONS
Q.1. Determine whether the system having input x (n) and output y (n) and described by
n
relationship : y (n) = ∑ x (k + 2) is (i) memoryless, (ii) stable, (iii)causal (iv)
k =-∞
linear and (v) time invariant. (5)

Ans:
y(n) = ∑ x(k + 2)
k = -∞
(i) Not memoryless - as y(n) depends on past values of input from x(-∞) to x(n-1)
(assuming)n > 0)
(ii) Unstable- since if x (n) ≤ M, then y(n) goes to ∞ for any n.
(iii) Non-causal - as y(n) depends on x(n+1) as well as x(n+2).
(iv) Linear - the principle of superposition applies (due to ∑ operation)
(v) Time – invariant - a time-shift in input results in corresponding time-shift in
output.

Q.2. Determine whether the signal x (t) described by


x (t) = exp [- at] u (t), a > 0 is a power signal or energy signal or neither. (5)

Ans:
-at
x(t) = e u(t), a > 0
x(t) is a non-periodic signal.
+∞ ∞ ∞
- at - at
Energy E = ∫ x2(t) dt = ∫ e 2 dt = e 2 = 1 (finite, positive)
-∞ 0 -2a 2a
0
The energy is finite and deterministic.
x(t) is an energy signal.

Q.3. Determine the even and odd parts of the signal x (t) given by
A e - α t t>0
x (t) = 
0 t<0

-αt
x(t) = Ae t>0
0 t<0 (5)

Ans:
Assumption : α > 0, A > 0, -∞ < t < ∞

Even part xe(t) = x(t) + x(-t)


2
Odd part xo(t) = x(t) - x(-t)
2

21
AE06/AC04/AT04 SIGNALS & SYSTEMS

x(t)
-αt
A Ae

t
0
x(-t)
+αt
Ae A

t
0
xe(t)
A/2

0 t
xo(t)
A/2

t
0
-A/2

Q.4. Use one sided Laplace transform to determine the output y (t) of a system described by
d2y dy dy
+ 3 + 2y (t) = 0 where y (0 − ) = 3 and =1 (7)
dt 2 dt dt t =0−
Ans:

d2y + 3 dy + 2 y(t) = 0, y(0-) = 3, dy =1


2 -
dt dt dt t = 0

s2 Y(s) – s y(0) – dy + 3 [s Y(s) – y(0)] + 2 Y(s) = 0


dt t = 0

(s2 + 3s + 2) Y(s) = sy(0) + dy + 3 y(0)


dt t = 0

(s2 + 3s + 2) Y(s) = 3s + 1 + 9 = 3s + 10

Y(s) = 3s + 10 = 3s + 10
2
s + 3s + 2 (s + 1)(s + 2)

= A + B
s+1 s+2

22
AE06/AC04/AT04 SIGNALS & SYSTEMS

A = 3s + 10 = 7; B = 3s + 10 = -4
s + 2 s = -1 s + 1 s = -2

Y(s) = 7 - 4
s+1 s+2

y(t) = L-1 [Y(s)] = 7e-t – 4e-2t = e-t( 7 - 4e-t)

The output of the system is y(t) = e-t( 7 - 4e-t) u(t)

Q. 5. Obtain two different realizations of the system given by


y (n) - (a+b) y (n – 1) + aby (n – 2) = x (n).Also obtain its transfer function. (7)

Ans:

y(n) – (a + b) y(n-1) + ab y(n-2) = x(n)

Y(z) – (a+b) z-1 Y(z) + ab z-2 Y(z) = X(z)

Transfer function H(z) = Y(z) = 1


X(z) 1 – (a+b) z-1 + ab z-2

y(n) = x(n) + (a + b) y(n-1) - ab y(n-2)

Direct Form I/II realization Alternative Realisation

Q. 6. An LTI system has an impulse response h (t) = exp [ -at] u (t); when it is excited by
an input signal x (t), its output is y (t) = [exp (- bt) -exp (- ct)] u (t) Determine its
input x (t). (7)

Ans:
-at
h(t) = e u(t) for input x(t)
-bt -ct
Output y(t) = (e - e ) u(t)

L L L
h(t) H(s), y(t) Y(s), x(t) X(s)

23
AE06/AC04/AT04 SIGNALS & SYSTEMS

H(s) = 1 ; Y(s) = 1 - 1 = s+c–s–b = c-b


s+a s+b s+c (s + b)(s + c) (s + b)(s + c)

As H(s) = Y(s) , X(s) = Y(s)


X(s) H(s)

X(s) = (c - b)(s + a) = A + B
(s + b)(s + c) s+b s+c

A = (c - b)(s + a) = (c – b)(-b + a) = a - b
(s + c) s = -b (-b + c)

B= (c - b)(s + a) = (c – b)(-c + a) = c - a
(s + b) s= -c (-c + b)

X(s) = a – b + c-a
s+b s+c
-bt -ct
x(t) = (a – b) e + (c – a) e
-bt -ct
The input x(t) = [(a – b) e + (c – a) e ] u(t)

Q.7. Write an expression for the waveform f (t ) shown in Fig. using only unit step function
and powers of t. (3)

Ans:

f(t) = E [ t u(t) – 2(t – T) u(t – T) + 2(t – 3T) u(t – 3T) – (t – 4T) u(t – 4T)]
T

Q.8. For f(t) of Q7, find and sketch f ′(t ) (prime denotes differentiation with respect to t). (3)

Ans:

f(t) = E [ t u(t) – 2(t – T) u(t – T) + 2(t – 3T) u(t – 3T) – (t – 4T) u(t – 4T)]
T

24
AE06/AC04/AT04 SIGNALS & SYSTEMS

f ΄(t)

E/T

t
0 T 2T 3T 4T

-E/T

f ΄(t) = E [u(t) – 2 u(t - T) + 2 u(t – 3T) –u(t – 4T)]


T

Q.9. Define a unit impulse function δ(t ) . (2)

Ans:
. Unit impulse function δ(t) is defined as:

δ(t) = 0, t ≠ 0
+∞
∫ δ(t) dt = 1
-∞

It can be viewed as the limit of a rectangular pulse of duration a and height 1/a when
a 0, as shown below.

3
Q.10. Sketch the function g (t ) =
3
(t − ∈)2 [u (t ) − u (t − ∈)] and show that (6)

g (t ) → δ(t ) as ∈→ 0 .

Ans:

g(t) As ε 0, duration 0, amplitude ∞


3/ε ε
∫ g(t) dt = 1
0
0 ε t

t
Q.11. Show that if the FT of x (t) is X ( jω) , then the FT of x   is a X ( jaω) . (6)
a
Ans:
FT
x(t) X(jω)

25
AE06/AC04/AT04 SIGNALS & SYSTEMS

FT
Let x t X1(jω) , then
a
+∞
X1(jω) = ∫ x t e-jωt dt Let t = α dt = a dα
-∞ a a
+∞
-jωaα
= ∫ x(α) e a dα if a> 0
-∞
+∞
-jωaα
- ∫ x(α) e a dα if a < 0
-∞ +∞
- α
Hence X1(jω) = a ∫ x(α) e jωa dα = a x (jωa)
-∞

Q.12. Solve, by using Laplace transforms, the following set of simultaneous differential
equations for x (t). (14)

Ans:
2 x ′(t ) + 4 x (t ) + y ′(t ) + 7 y(t ) = 5u (t )
x ′(t ) + x (t ) + y ′(t ) + 3y(t ) = 5δ(t )
The initial conditions are : x (0 − ) = y(0 − ) = 0 .
2 x΄(t) + 4 x(t) + y΄(t) + 7 y(t) = 5 u(t)
x΄(t) + x(t) + y΄(t) + 3 y(t) = 5 δ(t)
L L L L
x(t) X(s), x΄(t) s X(s), δ(t) 1, u(t) 1
s
(Given zero initial conditions)
2 sX(s) + 4 X(s) + sY(s) + 7 Y(s) = 5
s
sX(s) + X(s) + sY(s) + 3 Y(s) = 5

(2s + 4) X(s) + (s+7) Y(s) = 5


s
(s + 1) X(s) + (s+3) Y(s) = 5
X(s) = 5 s+7
S 3
5 s+3
2s+4 s+7
s+1 s+3
Or, X(s) = - 5s + 35 – 5 – 15/s
2s2 + 6s + 4s + 12 - s2 – 8s – 7

= - 5s2 + 30s –15 = -5 s2 + 6s – 3 = A + Bs+ C


2
s(s + 2s + 5) s s2 + 2s + 5 s s2+ 2s +5

Then A (s2+ 2s +5) + B s2 +Cs = -5(s2 + 6s – 3)

∴ A +B = -5
2A + C = -30

26
AE06/AC04/AT04 SIGNALS & SYSTEMS

5A =15

Thus A = 3, B = -8, C = -36 and we can write

X(s) = 3 – 8 s +1 – 14 2
2 2
s (s + 1) + 2 (s + 1)2 + 22
-t -t
x(t) = (3 – 8 e cos 2t – 14 e sin 2t) u(t)

Q.13. Find the Laplace transform of t sin ω0 t u (t ) . (6)

Ans:
L
sin (ω0t) ω0
s2 + ω02
L
Using t f(t) - d [F(s)],
ds

L [ t sin (ω0t) u(t) ] = - d ω0


ds s2 + ω02

= 0 - ω0(2s) = 2ω 0 s
(s2 + ω02)2 (s + ω02)2
2

s−2
Q.14. Find the inverse Laplace transform of . (8)
s(s + 1)3
Ans:

F(s) = s-2 = A + B + C + D
s(s+1)3 s s+1 (s+1)2 (s+1)3

A= s-2 = -2 A(s+1)3 + Bs(s+1)2 + Cs(s+1) + Ds = s-2


3
(s+1) s=0
s3 : A+B = 0 B=2
D= s-2 = 3
s s = -1 s2 : 3A + 2B + C = 0
C=2

A = -2 D=3

F(s) = -2 + 2 + 2 + 3
s s+1 (s+1)2 (s+1)3
-t -t -t
f(t) = -2 + 2 e + 2 t e + 3 t2 e
2
-t
f(t) = [-2 + e ( 3 t2 + 2t + 2 ) ] u(t)
2

27
AE06/AC04/AT04 SIGNALS & SYSTEMS

Q.15. Show that the difference equation y(n ) − αy(n − 1) = −αx (n ) + x (n − 1) represents an
all-pass transfer function. What is (are) the condition(s) on α for the system to be
stable? (8)

Ans:
y(n) – α y(n-1) = - α x(n) + x(n-1)

Y(z) – α z-1 Y(z) = - α X(z) + z-1 X(z)

(1-α z-1) Y(z) = (-α + z-1) X(z)

H(z) = Y(z) = -α + z-1 = 1- α z


X(z) 1- α z-1 z- α

Zero : z = 1 As poles and zeros have reciprocal values, the transfer function
α represents an all pass filter system.

Pole : z = α

Condition for stability of the system :

For stability, the pole at z = α must be inside the unit circle, i.e. α < 1.

Q.16. Give a recursive realization of the transfer function H (z ) = 1 + z −1 + z −2 + z −3 (6)

Ans:

H(z) = 1 + z-1 + z-2 + z-3 = 1 – z –4 Geometric series of 4 terms


–1
1–z First term = 1, Common ratio = z –1
As H(z) = Y(z) , we can write
X(z)
(1 – z –1) Y(z) = (1 – z –4) X(z) or Y(z) = X(z) (1 – z –4) = W (z)(1 – z –4)
(1 – z –1)
The realization of the system is shown below.

Q.17 Determine the z-transform of x 1 (n ) = α n u (n ) and x 2 (n ) = −α n u (− n − 1) and


indicate their regions of convergence. (6)

28
AE06/AC04/AT04 SIGNALS & SYSTEMS

Ans:

x1(n) = αn u(n) and x2(n) = -αn u(-n-1)

X1(z) = 1 RoC αz-1 < 1 i.e., z > α


1-αz -1

-1
X2(z) = ∑ - αn z-n
n=-∞

= - ∑ α-n zn = -( α-1z + α-2z2 + α-3z3 + ………)
n=1

= - α-1z ( 1 + α-1z + α-2z2 + ……..)

= - α-1z = z = 1 ; RoC α -1 z < 1 i.e., z < α


1- α-1z z-α 1 - α z-1

Q.18. Determine the sequence h (n ) whose z-transform is


1
H (z ) = , r <1. (6)
1 − 2 r cos θ z −1 + r 2 z − 2

Ans:

. H(z) = 1 , r < 1


-1 -2
1-2r cosθ z + r z
2

= 1 , r < 1
-
(1-r ejθ z-1) (1-r e jθ z -1)

= A + B = r < 1
-
(1-r e z ) (1-r e jθz -1)
jθ -1

where A= 1 = 1
(1-r ejθ z-1) jθ
r e z =1 -1
1- e -j2θ

B= 1 = 1
jθ -jθ
(1-r e z-1) r e z =1 -1
1- ej2θ

∴ h(n) = 1 ( r ejθ )n + 1 ( r e-jθ )n


1- e-2jθ 1- e2jθ
 e j nθ e − jnθ 
∴h(n) = rn  − j 2θ
+ j 2θ
 u(n)
1 − e 1− e 
 

n j(n + 1)θ
=r e - e - j(n + 1)θ u (n)
jθ -jθ
e -e

29
AE06/AC04/AT04 SIGNALS & SYSTEMS

= rn sin(n+1) θ u (n)
sinθ
1
Q.19. Let the Z- transform of x(n) be X(z).Show that the z-transform of x (-n) is X  . (2)
z
Ans:
z
x(n) X(z) Let y(n) = x(-n)
∞ ∞ ∞
-n +r -1 -1
Then Y(z) = ∑ x(-n)z = ∑ x(r) z = ∑ x(r) (z ) -1 = X (z )
n = -∞ r = -∞ r = -∞

 2πn 
Q.20. Find the energy content in the signal x (n ) = e −n 10 sin  . (7)
 4 
Ans:
- n
x(n) = e 0.1 sin 2πn
4
+∞ +∞ 2
-0.2 n
Energy content E = ∑ x (n) = ∑ e
2
sin 2πn
n=-∞ n=-∞ 4
+∞
E = ∑ e-2n sin2 nπ
n=-∞ 2
+∞
E = ∑ e-2n 1-cosnπ
n=-∞ 2
+∞
= 1 ∑ e-2n [1 – (-1) n]
2 n=-∞

Now 1 – (-1) n = 2 for n odd


0 for n even
∞ ∞
Also Let n = 2r +1 ; then E = ∑ e-.2(2r +1 ) = ∑ e-.4r e- .2
r=-∞ r=-∞
∞ ∞
= e-..2 ∑ e-.4r + ∑ e.4r The second term in brackets goes to infinity . Hence
r=0 r=1 E is infinite.

Q.21. Sketch the odd part of the signal shown in Fig. (3)

30
AE06/AC04/AT04 SIGNALS & SYSTEMS

Ans:
Odd part xo(t) = x(t) – x(-t)
2

Q.22. A linear system H has an input-output pair as shown in Fig. Determine whether the
system is causal and time-invariant. (4)

Ans:
System is non-causal the output y(t) exists at t = 0 when input x(t) starts only at
t = +1.
System is time-varying the expression for y(t) = [ u (t) – u (t-1)(t –1) + u (t –3)

(t – 3) – u (t-3) ] shows that the system H has time varying parameters.

Q.23. Determine whether the system characterized by the differential equation


d 2 y(t ) dy(t )
− + 2 y(t ) = x (t ) is stable or not. (4)
2 dt
dt

Ans:
d2y(t) - dy(t) +2y(t) = x(t)
dt2 dt
L L
y(t) Y(s); x(t) X(s); Zero initial conditions

s2 Y(s) – sY(s) + 2Y(s) = X(s)


31
AE06/AC04/AT04 SIGNALS & SYSTEMS

System transfer function Y(s) = 1 whose poles are in the right half plane.
X(s) s2 – s + 2
Hence the system is not stable.
t
Q.24 Determine whether the system y(t ) = ∫ x (τ) dτ is invertible. (5)
−∞
Ans:
t
y(t) = ∫ x(τ) dτ
-∞

Condition for invertibility: H-1 H = I (Identity operator)

H Integration
H-1 Differentiation

x(t) y(t) = H{x(t)}

H-1{y(t)} = H-1 H{x(t)} = x(t)

The system is invertible.

Q.25 Find the impulse response of a system characterized by the differential equation
y′(t ) + a y(t ) = x (t ) . (5)

Ans:

y΄(t) + a y(t) = x(t)


L L L
x(t) X(s), y(t) Y(s), h(t) H(s)

sY(s) + aY(s) = X(s), assuming zero initial conditions

H(s) = Y(s) = 1
X(s) s+a
-at
The impulse response of the system is h(t) = e u(t)

Q.26. Compute the Laplace transform of the signal y(t ) = (1 + 0.5 sin t ) sin 1000t . (4)

Ans:
y(t) = (1 + 0.5 sint) sin1000t
= sin 1000t + 0.5 sint sin 1000t

= sin 1000t + 0.5 cos 999t – cos 1001t


2
= sin 1000t + 0.25 cos 999t – 0.25 cos 1001t

Y(s) = 1000 + 0.25 s - 0.25 s


2 2
s + 1000 s + 9992
2
s + 10012
2

32
AE06/AC04/AT04 SIGNALS & SYSTEMS

Q.27. Determine Fourier Transform F(ω) of the signal f (t ) = e −αt cos(ωt + θ) and
determine the value of F(ω) . (7)

Ans:

We assume f(t) = e-αt cos(ωt + θ) u (t) because otherwise FT does not exist
FT +∞
f(t) F(ω) = ∫ e-αt ej(ωt + θ) + e-j(ωt + θ) e-jωt dt
2
+∞
F(ω) = 1 ∫ [e-αt e-jωt ejωt + jθ + e-αt e-jωt e-jωt – jθ] dt
2
+∞
= 1 ∫ [e-αt + jθ + e– jθ e– (α + 2jω)t] dt
2
+∞ − (α + 2 jω ) t ω
1 jθ e −αt − jθ e
F(ω ) = e +e
2 −α − (α + 2 jω ) 0
0
1 1 jθ 1
= e + e − jθ
2α α + 2 jω

F(ω) = 1 (α + 2jω) ejθ + α e– jθ


2 α (α + 2jω)

= 1 2α cos θ + 2jωejθ
2 α (α + 2jω)

= α cos θ + j ω cos θ – j ω sin θ


α (α + 2jω)

F(ω) 2 = α2 cos2θ+ ω2 –2αω sinθ + cosθ

α2 (α2 + 4ω2)

ω 2 + α2 cos 2θ – αω sin2 θ
=
α2 (α2 + 4ω2)

Q.28. Determine the impulse response h(t) and sketch the magnitude and phase
response of the system described by the transfer function (14)
s 2 + ωo2
H(s ) = .
2 ωo 2
s + s + ωo
Q
Ans:
H(s) = s 2 + ω0 2

s2 + ω0 s + ω02
Q
33
AE06/AC04/AT04 SIGNALS & SYSTEMS

H(jω) = (jω)2 + ω02 = ω02 - ω2


(jω) + ω0 (jω) + ω02
2
ω0 - ω2 + j ω ω0
2

Q Q

H(jω) = ω02 - ω2
1/ 2
(ω - ω2)2 + ω2 ω0 2
0
2

Q
Arg H(jω) = - tan-1 ω ω0
Q
ω02 - ω2

ω H(jω) Arg H(jω)


0 1 0
∞ 1 0
ω0- 0 - π/2
ω0+ 0 + π/2

H(jω)
Magnitude
1

0 ω0 ω
arg H(jω)
Phase
+ π/2

0 ω0 ω

- π/2

Q.29. Using the convolution sum, determine the output of the digital system shown in
Fig. below.
Assume that the input sequence is {x (n )} = {3, − 1, 3}and that the system is

initially at rest. n=0 (5)

Ans:
x(n) = { 3, -1, 3 }, system at rest initially (zero initial conditions)

n=0
34
AE06/AC04/AT04 SIGNALS & SYSTEMS

x(n) = 3δ(n) - δ(n-1) + 3δ(n-2)

X(z) = 3 - z-1 + 3z-2

Digital system: y(n) = x(n) + 1 y(n-1)


2
Y(z) = X(z) = 3 - z-1 + 3z-2 = -10 -6 z-1 + 13
1 – 1 z-1 1 – 1 z-1 1 – 1 z-1
2 2 2
by partial fraction expansion.
n
1
Hence y(n)= -10 δ (n) – 6 δ (n-1) + 13  u (n )
2

Q.30. Find the z-transform of the digital signal obtained by sampling the analog signal
e −4 t sin 4 t u (t ) at intervals of 0.1 sec. (6)

Ans:
t
x(t) = e-4 sin 4t u(t), T = 0.1 s
n
x(n) = x(t nT) = x(0.1n) = ( e-0.4 ) sin(0.4n)
z α = e-0.4 = 0.6703, 1 = 1.4918
x(n) X(z) α
z
x(n) = sin Ωn u(n) z sin Ω Ω = 0.4 rad = 22.92˚
z2 –2z cos Ω + 1
sin Ω = 0.3894; cos Ω = 0.9211

z
αn x(n) X (z/α)

X(z) = 1.4918z (0.3894)


(1.4918)2 z2 – 2(1.4918)z(0.9211) + 1

X(z) = 0.5809z
2.2255 z2 – 2.7482z + 1

Q.31. An LTI system is given by the difference equation y(n ) + 2 y(n − 1) + y(n − 2 ) = x (n ) .
i. Determine the unit impulse response.
ii. Determine the response of the system to the input (3, -1, 3).

n=0 (4)
Ans:
y(n) + 2y(n-1) + y(n-2) = x(n)

Y(z) + 2z-1 Y(z) + z-2 Y(z) = X(z)

(1 + 2z-1 + z-2)Y(z) = X(z)

35
AE06/AC04/AT04 SIGNALS & SYSTEMS

(i). H(z) = Y(z) = 1 = 1 ( Binomial expansion)


X(z) 1 + 2z-1 + z-2 (1 + z-1)2

= 1- 2z-1 + 3 z-2 - 4 z-3 + 5 z-4 - 6 z-5 + 7 z-6 - ….( Binomial expansion)

h(n) = δ(n) - 2δ(n-1) + 3δ(n-2) -…..

= {1,-2,3,-4,5,-6,7,….} is the impulse response.

n=0
(ii). x(n) = { 3,-1,3 }

n=0

= 3δ(n) - δ(n-1) + 3δ(n-2)

X(z) = 3 - z-1 + 3z-2

Y(z) = X(z).H(z) = 3 - z-1 + 3z-2 = 3(1 + 2z-1 + z-2) – 7z-1


1 + 2z-1 + z-2 1 + 2z-1 + z-2

= 3 – 7 z-1
(1 + z-1)2

y(n) = 3δ(n) + 7nu(n) is the required response of the system.

Q.32. The signal x(t) shown below in Fig. is applied to the input of an

(i) ideal differentiator. (ii) ideal integrator.

Sketch the responses. (1+4=5)

x(t) = t u(t) – 3t u(t-1) + 2t u(t-1.5)

36
AE06/AC04/AT04 SIGNALS & SYSTEMS

Ans:
x(t)

(i) 0 < t < 1


1 1
y(t) = ∫ t dt = t2 1 = 0.5 (Nonlinear)
0 2 0
t
0 1 1.5 (ii) 1 < t < 1.5
dx(t ) t
dt y(t) = y(1) + ∫(3-2t)dt
1 1
t
= 0.5 + (3t – t2) = 0.5 + 3t – t2 – 3 + 1
1
0 t = 3t – t2 – 1.5 (Nonlinear)

For t =1: y(1) = 3 – 1 – 1.5 = 0.5

(iii) t ≥ 1.5 : y(1.5) = 4.5 – 2.25 –1.5 = 0.75


-2

∫x(t)dt

0.5

0 1 1.5 t

Q.33. Sketch the even and odd parts of (1+2+3=6)


(i) a unit impulse function (ii) a unit step function
(iii) a unit ramp function.

Ans:
Even part xe(t) = x(t) + x(-t)
2
Odd part xo(t) = x(t) - x(-t)
2

(i) unit impulse (ii) unit step (iii) unit ramp


function function function
37
AE06/AC04/AT04 SIGNALS & SYSTEMS

 πt  πt
Q.34. Sketch the function f (t ) = u  sin  − u  − sin . (3)
 T   T 
Ans:

f(t)
1 f(t) = 1 0 < t T, 2T < t 3 T1
-1 T< t 2T, …
….. -T 0 T ….. t 3 T < t <4T, ……

-1


Q.35. Under what conditions, will the system characterized by y (n ) = ∑e
k = no
− ak
x(n − k ) be

linear, time-invariant, causal, stable and memoryless? (5)

Ans:
y(n) is : linear and time invariant for all k
causal if n0 not less than 0.
stable if a > 0
memoryless if k = 0 only

Q.36. Let E denote the energy of the signal x (t). What is the energy of the signal
x (2t)? (2)

Ans:
Given that
∞ 2

E= ∫ x(t )
−∞
dt

∞ 2

∫ x(2t )
1
To find E = dt
−∞
∞ 2 ∞ 2
dr 1 E
Let 2t =r then E1 = ∫
−∞
x(r ) =
2 2
−∞
∫ x(r ) dr =
2

Q.37. x(n), h(n) and y(n) are, respectively, the input signal, unit impulse response and
output signal of a linear, time-invariant, causal system and it is given that
y(n − 2) = x (n − n1 ) * h (n − n 2 ), where * denotes convolution. Find the possible
sets of values of n1 and n 2 . (3)

Ans:

y(n-2) = x(n-n1) * h(n-n2)


-n -n
z-2 Y(z) = z 1 X(z) . z 2 H(z)
z-2 H(z) X (z) = z − ( n1 + n 2 ) X(z)H (z )
n1+n2 = 2

38
AE06/AC04/AT04 SIGNALS & SYSTEMS

Also, n1, n2 ≥ 0, as the system is causal. So, the possible sets of values for n1 and n2 are:
{n1, n2} = {(0,2),(1,1),(2,0)}

Q.38. Let h(n) be the impulse response of the LTI causal system described by the difference
equation y(n ) = a y(n − 1) + x (n ) and let h (n ) * h1 (n ) = δ(n ) . Find h1 (n ) . (4)

Ans:
y(n) = a y(n-1) + x(n) and h(n) * h1(n) = δ(n)

Y(z) = az-1 Y(z) + X(z) and H(z) H1(z) =1


H(z) = Y(z) = 1 and H1(z) = 1
X(z) 1-az-1 H(z)

H1(z) = 1-az-1 or h1(n) = δ(n) – a δ(n-1)

Q.39. Determine the Fourier series expansion of the waveform f (t) shown below in terms of
sines and cosines. Sketch the magnitude and phase spectra. (10+2+2=14)

Ans:

Define g(t) = f(t) +1. Then the plot of g(t) is as shown , below and,

ω = 2π/2π = 1
because T =2π

g(t) = 0 - π< t < - π/2


2 - π/2< t < π/2
0 π/2< t < π


Let g(t) = a0 + Σ (an cos nt + bn sin nt)
n=1
Then a0 = average value of f(t) =1

39
AE06/AC04/AT04 SIGNALS & SYSTEMS

π /2
2 2 sin nt π/2
an =
2π ∫π 2 cos ntdt
− /2
=
π n
= 2 /n π . 2sin n π/2

-π/2
= 4 /n π . sin n π/2
= 0 if n= 2,4,6 ……
4 /n π if n= 1,5,9 ……
- 4 /n π if n= 3,7,11……
π /2
2 4 cos nt π/2
Also, bn =
2π ∫π 2 sin ntdt
− /2
=
π n
= 4 /n π[ cos n π /2 - cos n π /2] = 0

-π/2
Thus, we have f(t) = -1 + g(t)
4cost 4cos 3t 4 cos 5t
= − + − .......
π 3π 5π
= 4/ π { cost – cos3t /3 + cos5t/5 …..}

spectra : (Magnitude) X π/4


1

1/3
1/5
1/7
-7 -6 -5 -4 -3 -2 -1 0 1 2 3 4 5 6 7 a

Phase

-7 -3 3 7

Q.40. Show that if the Fourier Transform (FT) of x (t) is X(ω) , then (3)
 dx (t ) 
FT  = jω X(ω) .
 dt 

Ans:
FT
x(t) X(jω) or X(ω)
+∞
i.e., x(t) = 1 ∫ X(jω) ejωt dω
2π -∞
+∞
d [x(t)] = 1 ∫ X(jω) jω ejωt dω
dt 2π - ∞

d [x(t)] FT jω X(jω)
dt
1 
Q.41. Show, by any method, that FT   = π δ (ω ) . (2)
2
40
AE06/AC04/AT04 SIGNALS & SYSTEMS

Ans:
+∞
x(t) = 1 ∫ X(jω) ejωt dω
2π - ∞
+∞
x(t) = 1 ∫ π δ(ω) ejωt dω = 1 X(jω) = π δ(ω)
2π - ∞ 2

1 FT π δ(ω)
2

Q.42 Find the unit impulse response, h(t), of the system characterized by the relationship :
t
y(t ) = ∫ x (τ ) dτ . (3)
−∞

Ans:
t
y(t) = ∫ δ (τ) dτ = 1, t ≥ 0 = u(t)
-∞ 0, otherwise

Q.43. Using the results of parts (a) and (b), or otherwise, determine the frequency response
of the system of part (c). (6)

Ans:
As shown in the figure, u(t) = 1/2 + x(t)
where x(t) = 0.5, t >0
-0.5, t <0
∴ dx/dt = δ (t) By (a) FT[δ (t)] = jωX(ω)
∴ X(ω) = 1/jω. Also FT[1/2] = πδ (ω)
Therefore FT [u(t)] = H(jω)= π√(ω) + 1/j ω.

( )
Q.44. Let X e jω denote the Fourier Transform of the signal x (n) shown below .(2+2+3+5+2=14)

41
AE06/AC04/AT04 SIGNALS & SYSTEMS

( )
Without explicitly finding out X e jω , find the following :-
π
(i) X (1) (ii) ∫ ( )
X e jω dω
−π
(iii) X(-1) (iv) the sequence y(n) whose Fourier
( )
Transform is the real part of X e jω .
π
(v) ∫ ( ) 2 dω .
X e jω
−π

Ans: ∞
jω -jωn
X(e ) = ∑ x(n) e
n = -∞
+∞
j0
(i) X(1) = X(e ) = ∑ x(n) = -1 + 1 + 2 + 1 + 1 + 2 + 1 –1 = 6
-∞

+π π
jω jωn jω
(ii) x(n) = 1 ∫ X(e ) e dω ; ∫ X(e ) dω = 2π x(0) = 4π
2π -π -π
+∞

(iii) X(-1) = X(e ) = ∑ x(n) (-1)n = 1+ 0-1+2-1+0-1+2-1+0+1 =2
n = -∞

(iv) Real part X(e ) xe(n) = x(n) + x(-n)
2
y(n) = xe(n) = 0, n < -7, n > 7
y(7) = 1 x(7) = -1 = y(-7)
2 2
y(6) = 1 x(6) = 0 = y(-6)
2
y(5) = 1 x(5) = 1 = y(-5)
2 2
y(4) = 1 x(4) = 2 = y(-4)
2
y(3) = 1[x(3) + x(-3)] = 0 = y(-3)
2
y(2) = 1[x(2) + x(-2)] = 0 = y(-2)
2
y(1) = 1[y(1) + y(-1)] = 1 = y(-1)
2
y(0) = 1[ y(0) + y(0)] = 2
2
(v) Parseval’s theorem:
π 2 ∞ 2

∫ X(e ) dω = 2π ∑ x(n) = 2π(1 + 1 + 4 +1 + 1 + 4 + 1 + 1) = 28π
-π n = -∞

Q.45 If the z-transform of x (n) is X(z) with ROC denoted by R x , find the

42
AE06/AC04/AT04 SIGNALS & SYSTEMS

n
z-transform of y(n ) = ∑ x (k ) and its ROC. (4)
k = −∞

Ans:
z
x(n) X(z), RoC Rx
n 0 ∞
y(n) = ∑ x(k) = ∑ x(n-k) = ∑ x(n-k)
k = -∞ k=∞ k=0

Y(z) = X(z) ∑ z-k = X(z) , RoC at least Rx ∩ (z > 1)
k=0 1 - z-1

Geometric series

Q.46 (i) x (n) is a real right-sided sequence having a z-transform X(z). X(z) has two
poles, one of which is at a e jφ and two zeros, one of which is at r e − jθ . It is also
known that ∑ x (n ) = 1 . Determine X(z) as a ratio of polynomials in z −1 . (6)
(ii) If a = 1 , r = 2, θ = φ = π 4 in part (b) (i), determine the magnitude of X(z) on the
2
unit circle. (4)

Ans:
z
. (i) x(n) : real, right-sided sequence X(z)
-jθ jθ
X(z) = K (z- re )(z- re ) ; ∑x(n) = X(1) = 1
jΦ -jΦ
(z- ae )( z- ae )
jθ -jθ
= K z2 –zr (e +e ) + r2
jΦ jΦ
z2 –za (e + e ) + a2

= K 1 – 2r cosθ z-1 + r2 z-2 = K. N(z-1)


1 – 2a cosΦz-1 + a2 z-2 D(z-1)

where K. 1 – 2r cosθ + r2 = X(1) = 1


1 – 2a cosΦ + a2

i.e., K = 1– 2a cosΦ + a2
1 – 2r cosθ + r2

(ii) a = ½, r = 2, θ = Φ = π/4 ; K = 1 – 2(½).(1/√2) + ¼ = 0.25


1 – 2(2) (1/√2) + 4

X(z) = (0.25) . 1 – 2(2) (1/√2) z-1 + 4z-2


1 – 2(½).(1/√2) z-1 + ¼ z-2

43
AE06/AC04/AT04 SIGNALS & SYSTEMS
jω -jω - jω
= (0.25) 1 - 2√2 z-1 + 4z-2 X(e ) = (0.25) 1 - 2√ 2 e +4e2
-jω - jω
1 – (1/√2) z-1 + ¼ z-2 1 – (1/√2) e +¼e2

jω -jω
= - 2√ 2 + e + 4 e
jω -jω
-2√2+ 4e + e

X(e ) =1

Q.47 Determine, by any method, the output y(t) of an LTI system whose impulse
response h(t) is of the form shown in fig(a). to the periodic excitation x(t) as
shown in fig(b). (14)

Ans:

Fig(a) Fig(b)

1- e -s
h(t) = u(t) – u(t-1) => H(s) =
s
First period of x(t) , xT(t) = 2t [u(t) – u(t- ½) ]

= 2[ t u(t) – (t-1/2) u(t-1/2) –1/2 u(t-1/2)]


∴ XT(s) = 2[1/s2 – e-s/2 / s2 – 1/2 e-s/2 / s ]
X (s) = XT(s) / 1 – e-s/2
1 − e−s 1  1− e −s / 2
− 0.5se − s / 2 
Y(s) = . 2  
s 1 − e −s / 2  s2 

=
2
s 3
(1 + e−s / 2 )[1 − e−s / 2 − 0.5 s e−s / 2 ]
=
2
(1 − e −s
− 0.5s (e − s / 2 + e − s ) )
s3
1 − e −s e −s / 2 + e −s
=2 −
s3 s2
 1  1
Therefore y(t) = t u(t) – (t-1)2 u(t-1) –  t − u  t +  − ( t − 1) u ( t − 1)
2
 2  2
This gives y (t) = t2 0< t < 1/2
t2 –t +1/2 1/2 < t < 1
1/2 t >1

44
AE06/AC04/AT04 SIGNALS & SYSTEMS

(not to scale)
s 2 + 3s + 1
Q.48 Obtain the time function f(t) whose Laplace Transform is F(s ) = . (14)
(s + 1)3 (s + 2)2
Ans:

F(s) = s2+3s+1 = A + B + C + D + E
(s+1)3(s+2)2 (s+1) (s+1)2 (s+1)3 (s+2) (s+2)2

A(s+2)2(s+1)2 + B(s+2)2(s+1) + C(s+2)2 + D(s+1)3(s+2) +E(s+1)3 = s2+3s+1

C = s2+3s+1 = 1-3+1 = -1 C = -1
(s+2)2 s= -1 1

E = s2+3s+1 = 4-6+1 = 1 E=1


3
(s+1) s= -2 -1

A(s2+3s+2)2 + B(s2+4s+4)(s+1) + C(s2+4s+4) + D(s3+3s2+3s+1)(s+2) + E(s3+3s2+3s+1)


= s2+3s+1
A(s4+6s3+13s2+12s+4) + B(s3+5s2+8s+4) + C(s2+4s+4) + D(s4+5s3+9s2+7s+2) +
E(s3+3s2+3s+1) = s2+3s+1
s4 : A+D = 0
3
s : 6A+ B+ 5D +E = 0 ; A+B+1 = 0 as 5(A+D) = 0, E = 1
s2 : 13A+5B+C+9D+3E = 1 ; 4A+5B+1 = 0 as 9(A+D) = 0, C = -1, E = 1
s1 : 12A+8B+4C+7D+3E = 3 ; 5A+8B-4 = 0 as 7(A+D) = 0, C = -1, E = 1
s0 : 4A+4B+4C+2D+E = 1
A+B = -1 ; 4(A+B)+B+1 = 0 or –4+B+1 = 0 or
B=3
A=-4
A = -1-3 = - 4

A+D = 0 or D = -A = 4 D=4

F(s) = - 4 + 3 + -1 + 4 + 1
(s+1) (s+1)2 (s+1)3 (s+2) (s+2)2

f(t) = L-1[F(s)] = - 4e-t + 3t e-t – t2 e-t + 4e-2 t + t e-2t = [e-t(-4 + 3t - t2) + e-2 t(4 + t)] u(t)
45
AE06/AC04/AT04 SIGNALS & SYSTEMS

f(t) = [e-t(-4 + 3t - t2) + e-2 t(4 + t)] u(t)

Q.49 Define the terms variance, co-variance and correlation coefficient as applied to
random variables. (6)

Ans:

Variance of a random variable X is defined as the second central moment


E[(X-µX)]n, n=2, where central moment is the moment of the difference
between a random variable X and its mean µX i.e.,
+∞
σX² = var [X] = ∫ (x- µX)² fx(x) dx
-∞
Co-variance of random variables X and Y is defined as the joint moment:
σXY = cov [XY] = E[{X-E[X]}{Y-E[Y]}] = E[XY]-µ Xµ Y
where µ X = E[X] and µ Y = E[Y].
Correlation coefficient ρXY of X and Y is defined as the co-variance of X and Y
normalized w.r.t σXσY :
ρXY = cov [XY] = σXY
σXσY σXσY

Q.50 Determine the total energy of the raised-cosine pulse x(t), shown in Fig.1 defined by:
(8)
 1 π π
 (cos ω + 1), − ≤ t ≤
x(t) =  2 ω ω .
 0, otherwise

t
π π

ω ω
Fig.1
Ans:
π
+∞ ω
1 3π
∫x ∫ 4 (cos ωt + 1) dt = 4ω units.
2 2
Energy E = ( t )dt =
−∞ − ωπ

FT
Q.51 State the sampling theorem, given x ( t ) ↔ X (ω) . For the spectrum of the continuous-time
signal, shown in Fig.2, consider the three cases f s = 2f x ; f s > 2f x ; f s < 2f x and draw the
spectra, indicating aliasing. (8)

46
AE06/AC04/AT04 SIGNALS & SYSTEMS

− fx fx
Fig.2
Ans:
FT
Sampling theorem: Given x ( t ) ↔ X (ω) , if X (ω) = 0 for ω > ωm , and if ωs > 2ωm ,

where sampling frequency ωs = , Ts = Sampling interval, then x(t) is uniquely
Ts
determined by its samples x ( nTs ) → where n = 0, ± 1, ± 2, ….. ( ωs > 2ω m ⇒
Nyquist rate.)

X δ (f )
f s X ( 0)

-fx 0 fx f f
fs = f x

-fx 0 fx f f
(2fx) f s > 2f x

Guardband

-fx 0 fx 2fx f f

overlap f s < 2f x
⇒ alia sin g
-fx 0 fx f f

Q.52 Consider a continuous-time signal x(t). (8)


FT
(i) Show that X ( t ) ↔ 2π x (−ω) , using duality (or similarity) property of FT s.
1
(ii) Find x(t) from X (ω) = , using the convolution property of FTs.
(1 + jω) 2
Ans:
+∞
1
X(ω)e jωt dω
2π ω=∫−∞
(i) x(t) =

Using duality property of FTs, t ↔ ω ,

47
AE06/AC04/AT04 SIGNALS & SYSTEMS

+∞ +∞
1 1
∫ X ( t )e jtωdt , or , x ( −ω) = X ( t )e − jtω dt
2π t =∫−∞
x (ω) =
2π t =−∞
+∞ FT
− jtω
∴ 2π x (−ω) = ∫ X( t )e dt , i.e., X(t ) ↔ 2πx (−ω) .
−∞
1
(ii) Find x(t) from X (ω) = , using the convolution property of FTs.
(1 + jω) 2
1 1 1 FT 1
X (ω) = = . , and, e − t u ( t ) ↔ .
(1 + jω) 2 1 + jω 1 + jω 1 + jω
FT
Convolution property of FTs ⇒ x ( t ) = x1 ( t ) * x 2 ( t ) ↔ X (ω) = X1 (ω)X 2 (ω) .
+∞
−τ 1, 0 < τ < t > 0
∴ x(t) = ∫e u (τ)e −( t −τ) u ( t − τ)dτ . u (τ)u ( t − τ) = 
−∞ 0, t < 0.
= t.e − t , t > 0
∴ x(t) = t e −t u (t ) .

Q.53 Find the difference equation describing the system represented by the block-diagram
shown in Fig.3, where D stands for unit delay. (8)

Σ Σ

1

2

1

4
Fig.3
Ans:
Intermediate variable f(n) between the summers:
1 1
f (n ) = x (n − 1) − f (n − 1) − f (n − 2)
2 4
y(n ) = 2 x (n ) + f (n ) , or, f (n ) = y(n ) − 2 x (n )
f ( n − 1) = y( n − 1) − 2 x ( n − 1)
f ( n − 2) = y ( n − 2) − 2 x ( n − 2)
1 1 1
y(n ) + y(n − 1) + y(n − 2) = 2 x (n ) + 2 x (n − 1) + x (n − 2) .
2 4 2

Q.54 For the simple continuous-time RC frequently-selective filter shown in Fig.4, obtain the
frequency response H(ω). Sketch its magnitude and phase for -∞ < ω < ∞. (8)

48
AE06/AC04/AT04 SIGNALS & SYSTEMS

Fig.4
Ans:
dy( t ) FT
KVL ⇒ x ( t ) = RC + y( t ) ↔ X(ω) = RCjωY(ω) + Y(ω)
dt
Y (ω) 1 1
or, H (ω) = = = .
X (ω) 1 + jωCR  ω
1 + j 
 ω0 
1
H (ω) =
2
 ω
1 +  
 ω0 
 ω
arg H(ω) = − tan −1  
 ω0 
π
magnitude phase spectrum
spectrum 2

3 dB
0 ω
π

2
- ω0 0 ω0 ω

Q.55 Consider the signal x ( t ) = e − t u ( t ) + e −2 t u ( t ) . Express its Laplace Transform in the


N (s)
form: X (s) = K. , K = system constant. Identify th region of convergence.
D(s)
Indicate poles and zeros in the s-plane. (8)

Ans:
L 1 1
x ( t ) = e − t u ( t ) + e − 2 t u ( t ) ↔ X (s ) = +
s +1 s + 2
 3
s + 
2s + 3
=1 
2 N(s)
X (s) = =K
(s + 1)(s + 2) (
s 2 + 3s + 2 D(s) )
, K = 2.

49
AE06/AC04/AT04 SIGNALS & SYSTEMS

3
1 Zero ⇒ s = − .
2
2 Poles ⇒ s = −1,−2.
ROC ⇒ R e {s} > −1.
R e {s} > −2.
∴ Common R o C is R e {s} > −1.

Q.56 Given input x(n) and impulse response h(n), as shown in Fig.5, evaluate
y(n ) = x (n ) * h (n ) , using DTFTs. (8)

Fig.5
Ans:
DTFT
y(n ) = x (n ) * h (n ) ↔ Y (e jΩ ).H (e jΩ ) .
( ) n = −1 n = 0 n =1 ( ) n = −1 n = 0 n =1
H e jΩ = e + jΩ + 1 + e − jΩ ; X e jΩ = ae + jΩ + a + ae − jΩ

∴ Y (e jΩ ) = a (e jΩ + 1 + e − jΩ ) = a (e − j2Ω + 2e jΩ + 3+ 2e − jΩ + e − j2Ω )
2

DTFT
As δ(n − n 0 ) ↔ e − jΩn o , y(n) = aδ(n+2) + 2aδ(n+1) + 3aδ(n) + 2aδ(n-1) + aδ(n-2).
y(n) = {a, 2a, 3a, 2a, a}

n=0

Q.57 Determine the inverse DTFT, by partial fraction expansion, of


( ) = e− j2Ω − 5e− jΩ + 6 .
Xe jΩ 6
(8)

Ans:
( )
X e jΩ =
6
=
(
6
)( )
−2
= +
3
.
(e − jΩ 2
) (
−5e − jΩ
)
+6 e − jΩ
− 3 e − jΩ
− 2 1 − jΩ
1− e
3
1 − jΩ
1− e
2
1
n
1
n  1 n
1 
n
∴ x (1n ) = −2  u (n ) + 3  u (n ) = − 2  + 3   u (n ) .
 3 2   3   2  

50
AE06/AC04/AT04 SIGNALS & SYSTEMS

Q.58 State the initial-value and final-value theorems of Laplace Transforms. Compute the
L 3s + 4
initial-value and final-values for x ( t ) ↔ X(s) , where x (s) = . (8)
s(s + 1)(s + 2) 2
Ans:
Initial-value theorem: If f(t) and its first derivative are Laplace transformable, then
the initial value of f(t) is: f (0 + ) = lim f ( t ) = lim sF(s) .
t →0 + s →∞
Final-value theorem: If f(t) and its first derivative are Laplace transformable, and
f(t) is not a periodic function, then the final value of f(t) is: lim f ( t ) = lim sF(s) .
t →∞ s →0
 4
3 + 
Initial value ⇒ x (0 + ) = lim sX(s) = lim  s
= 0.
s →∞ s →∞  1
1 + (s + 2 )
2
 s
Final value ⇒ lim x ( t ) = lim sX(s) = lim
(3s + 4) = 1 .
t →∞ s →0 s →0 (s + 1)(s + 2 )2

Q.59 Find, by Laplace Transform method, the output y(t) of the system described by the
dy( t )
differential equation: + 5 y( t ) = x ( t ) where input x ( t ) = 3e −2 t u ( t ) and the initial
dt
condition is y(0) = -2. (8)

Ans:
dy( t )
+ 5 y( t ) = 3e − 2 t u ( t ), y(0) = -2.
dt
L L 1
y( t ) ↔ Y(s), u ( t ), e − 2 t ↔ .
s+2
3
∴ sY (s) − y(0 + ) + 5Y (s) = .
s+2
3 −2 A B 2
Y (s) = + = + − .
(s + 2)(s + 5) (s + 5) s + 2 s + 5 s + 5
1 3 3
= − . A= s = −2 = 1
s+2 s+5 s+5
( )
∴ y( t ) = e − 2 t − 3e −5 t u ( t ) B=
3
s+2
s = −5 = −1 .

Q.60 An LTI system is characterised by the difference equation: x(n – 2) – 9x(n – 1) + 18x(n)
= 0 with initial conditions x(-1) = 1 and x(-2) = 9. Find x(n) by using z-transform and
state the properties of z-transform used in your calculation. (8)

Ans:
x(n – 2) -9x(n – 1) + 18x(n) = 0
By using
z
x (n − n 0 ) ↔ z −n 0 X (z) + x (−n 0 ) + z −1x (−n 0 + 1) + z − 2 x (−n 0 + 2) + ..... + z −( n 0 −1) x (−1)

51
AE06/AC04/AT04 SIGNALS & SYSTEMS

We get [ x ( −2) + x ( −1) z −1 + z −2 X( z)] − 9[ x ( −1) + z −1X ( z)] + 18X ( z) = 0 .


123 123 123
9 1 1
 
− z −1 z −1  1 2 
X (z) = =  − .
 1 −1 1 − 2  18  1 − 1 z −1 1 − 1 z −1 
181 − z + z   
 2 18   6 3 
1  1  1 
n −1 n −1
∴ x (n ) =   − 2   u (n − 1) .
18  16   3  

z z2 + z
Q.61 Determine the discrete-time sequence x(n), given that x (n ) ↔ X (z) = .
z 3 − 3z 2 + 3z − 1
(8)
Ans: Assume that x(n) is casual. Then

z 3 − 3z 2 + 3z − 1 z 2 + z z −1 + 4z −2 + 9z −3 + 16z −4 + ......
z 2 − 3z + 3 − z −1
4z − 3 + z −1
4z − 12 + 12z −1 − 4z −2
9 − 11z −1 + 4z − 2
9 − 27 z −1 + 27 z −2 − 9z −3
16z −1 − 23z − 2 + 9z − 3
16z −1 − 48z −2 + 48z −3 − 16z −4
25z − 2 − 39z −3 + 16z − 4
∴ X ( y) = z −1 + 4z −2 + 9z −3 + 16z −4 + ..... δ(n − n ) ←→
z
z−n 0 
 0 
∴ x ( n ) = δ( n − 1) + 4δ(n − 2) + 9δ( n − 3) + 16δ( n − 3) + .....
x(n) = {0, 1, 4, 9, 16, …..}

n=0

Q.62 Explain the meaning of the following terms with respect to random variables/processes:
(i) Wide-sense stationary process.
(ii) Ergodic process.
(iii) White noise.
(iv) Cross power spectral density. (8)

Ans:
(i) Wide-sense stationary process.
For stationary processes, means and variances are independent of time, and
covariance depends only on the time-difference if in addition, the N-fold joint p.d.f.
depends on the time origin, such a random process is called wide- sense stationary
process.
(ii) Ergodic process.
Ergodic process is one in which time and ensemble averages are interchangeable.

52
AE06/AC04/AT04 SIGNALS & SYSTEMS

For ergodic processes, all time and ensemble averages are interchangeable, not just
the mean, variance and autocorrelation function.
(iii) White noise.
White noise is an idealised form of noise, the power spectral density of which is
independent of frequency. “White” is in parlance with white light that contains all
frequencies within the visible band of electromagnetic radiation..
(iv) Cross power spectral density.
Cross power spectral density of two stationary random processes is defined as the
FT of their cross-correlation function
FT
R xy (τ) ↔ Sxy (f ), where, R xy (τ) = E{X ( t ).Y( t + τ)} .

Q.63 A random variable X is characterised by probability density function shown in Fig.6:


 x
1 − , 0 ≤ x ≤ 2
f x (x) =  2 f X (x )
 0, otherwise
Compute its: Probability distribution function;
Probability in the range 0.5< x ≤1.5;
Mean value between 0 ≤ x ≤ 2; and
Mean-square value E(x2).
Fig.6 (8)
Ans:
+∞ x
 α x2
p.d.f. f X ( x ) = ∫X
f ( α ) d α = ∫ 2 
 1 − d α = x −
4
, 0< x ≤ 2.
−∞ 0
1
Probability = f X (1.5) − f X (0.5) = .
2
(0.5 < X ≤ 1.5)
2
 x 2
Mean value m X = ∫ x 1 − dx =
0 
2 3

[ ]
2
 x 2
Mean-squared value E x = ∫ x 2 1 − dx = .
2

0  2 3

Q.64 Determine the fundamental frequency of the signal


− j 4πn j 3πn
x[ n] = e 3
+e 8
.
Ans:
− j 4πn − j 3πn − j 32πn − j 9πn

x (n) = e 3
+e 8
=e 24
+e 24

1
Fundamental Frequency = .
24
t3

Q.65 A CT system is described by y (t ) = ∫ x(λ )dλ . Find if the system is time


−∞
invariant and stable. (6)

Ans:

53
AE06/AC04/AT04 SIGNALS & SYSTEMS

t
3
y(t) =
−∞
∫ x(λ )dλ .
Let x(t) be shifted by t0 then the corresponding output yi(t) will be
t t
−t0
3 3
yi(t) = ∫ x (λ − t
−∞
0 ) dλ = ∫ x(λ ′)dλ ′ where λ ′ = λ − t
−∞
0 .

Original output shifted by t0 sec is


t
−t0
3
y0(t) =
−∞
∫ x ( λ ) dλ
Hence the system is time-invariant.
If x(t) is bounded, output will be bounded. Hence the system is stable.
Q.66 Let x(t ) be a real signal and x(t ) = x1 (t ) + x 2 (t ) . Find a condition so that
∞ ∞ ∞

∫ x(t ) dt = ∫ x1 (t ) dt + ∫ x (t )
2 2 2
2 dt (6)
−∞ −∞ −∞

∞ 2

∫−∞ ( x1 (t ) + x2 (t ) ) dt

∫ x(t ) dt =
2
Ans:
−∞
∞ ∞ ∞ ∞

∫ x(t ) dt = ∫ x1 (t ) dt + ∫ x 2 (t ) dt + 2 ∫ x1 (t ) x 2 (t ) dt
2 2 2

−∞ −∞ −∞ −∞

The term 2 ∫ x1 (t ) x 2 (t ) dt will become zero if x1 (t ) is the even part and x 2 (t )
−∞

is the odd part of x(t ) or vice-versa. Then


∞ ∞ ∞

∫ x(t ) dt = ∫ x1 (t ) dt + ∫ x (t )
2 2 2
2 dt .
−∞ −∞ −∞

Q.67 If h1 [n] = δ [n] , h2 [n] = δ [n − 1] + 2δ [n − 2] , h3 [n] = δ [n + 1] + 2δ [n + 2]


are the impulse responses of three LTI systems, determine the impulse
response of the system shown in Fig.1.

h1 [n]
+

+
Fig. 1 h2 [n] h3 [n]

Ans: h1(n) = δ(n)


h2(n) = δ(n-1)+2δ(n-2)
h3(n) = δ(n+1)+2δ(n+2)
Impulse response of the system,
h(n) = h1(n) + h2(n)*h3(n)
= δ(n) + 2δ(n+1) + 5δ(n) + 2δ(n-1)
= 6δ(n) + 2δ(n+1) + 2δ(n-1).

54
AE06/AC04/AT04 SIGNALS & SYSTEMS

Q.68 Given that y (t ) = x(t ) ∗ h(t ) , determine x(at ) ∗ h(at ) in terms of y (t ) . If a is


real, for what values of a the system will be (i) causal, (ii) stable? (10)
Ans: y(t) = x(t)*h(t)
Thus, Y(s) = X(s)H(s)
Now x(at) has Laplace transform (1/a) X(s/a) . Similarly h(at) has Laplace
transform (1/a) H(s/a) . Thus
Laplace transform of x(at)*h(at) = (1/a2)X(s/a) H(s/a)
= (1/a) (1/a) Y(s/a)
= Laplace transform of (1/a)y(at)
Assuming the original system to be causal and stable,
(i) to maintain only causality, a can take any value,
(ii) to maintain stability, a > 0.
Q.69 One period of a continuous-time periodic signal x(t ) is as given below.
 t , −1 < t < 1
x (t ) =  (10)
 0, 1 < t < 2
Determine Fourier series coefficients of x(t ) , assuming its period to be 3.

 t , −1 < t < 1
Ans: x(t ) = 
 0, 1 < t < 2
1  0 1
 1  − t 2  0  t 2 1  1
a0 =
T ∫
−1 0

 − tdt + tdt  =
 T
  +  =
 2  −1  2  0  T

1   1  0 
1

∫ ∫ ∫
− jkω t − jkω t − jkω t
ak =  x(t )e 0 dt  =  − te 0 dt + te 0 dt 
T T  T −1 0 

1   e jk ω 0 e jk ω 0 − 1   e − jk ω 0 e − jk ω 0 − 1  
=  + +
  − + 
T   jk ω 0 k 2 ω 0 2   jk ω 0 k 2 ω 0 2  
 2  e jk ω 0 − e − jk ω 0   2  e jk ω 0 + e − jk ω 0
1   
=   + 2 2  − 1  
 k ω 0 
T 2j   k ω 0  2   
2  sin k ω 0 cos k ω 0 − 1 
=  + 
T  k ω 0 k 2ω 0
2

 sin 2 π k / T cos 2 π k / T − 1 
= +T 
 πk 2π 2 k 2 
1
a0 T =3 =
3
 sin 2 π k / 3 cos 2 π k / 3 − 1 
a k T =3 =  +3 
 πk 2π 2 k 2 

Q.70 Determine Fourier series coefficients of the same signal x(t ) as in Q69, but
now, assuming its period to be 6. What is the relationship between the
coefficients determined in Q69 & Q70? (6)
Ans:Let the period be T2 = 2T1 = 6. Following the above procedure, we get

55
AE06/AC04/AT04 SIGNALS & SYSTEMS

1 0 2
 4 2
a02 =
T2−2
∫ 0

 − tdt + tdt  =
 T2 T1
= = 2a01

1  
0 2


T2 − 2 0

ak 2 =  − te − jkω 0 t dt + te − jkω 0 t dt 

 sin 4 π k / T 2 cos 4 π k / T 2 − 1 
= 2 + T2 
 πk 4π 2 k 2 
 sin 2 π k / T cos 2 π k / T − 1 
= 2 1
+ T1 1
 = 2ak1
 π k 2 π 2 2
k 
Thus the Fourier coefficients are doubled when the period is doubled. The
function with higher period will have all the harmonics present in the lower
period function as even harmonics.

Q.71 The Fourier transform of a signal x(t) is described as


 0.5π − 1 < ω < 0
 ω , −1 < ω < 1 
X ( jω ) =  and arg X ( jω ) = − 0.5π 0 < ω < 1
 0, otherwise  0 otherwise

Determine whether x(t) is real or complex.

Ans:The magnitude response is symmetric and the phase response is anti-


symmetric. So x(t ) is real.
4
Q.72 Determine the inverse Fourier transform of X ( jω ) = 2 sin 2 ω using the
ω
convolution property of the Fourier transform. (4)

4 2  2 
Ans: X ( jω ) = sin 2 ω =  sin ω  sin ω  =P(jω).P(jω)
ω2 ω  ω 
Multiplication in the frequency domain is equivalent to convolution in the
time domain.
sin ω
Inverse Fourier transform of P(jω) = 2 is a pulse
ω
1, t ≤1
p (t ) = 
0, t > 1.
Therefore
2 − t , t ≤ 2
x (t ) = p(t ) ∗ p (t ) = 
0, t >2
Q.73 A system is described by the difference equation y[n] = x[n] + ay[n − 1] .
Find the impulse response of the inverse of this system. From the impulse
response, find the difference equation of the inverse system. (8)

Ans: y[n] = x[n] + ay[n − 1]


Y ( z) 1
H ( z) = =
X ( z ) 1 − az −1
For Inverse System:

56
AE06/AC04/AT04 SIGNALS & SYSTEMS

1
Transfer function H 1 ( z ) = = 1 − az −1
H ( z)
Impulse Response: h1(n) = 1δ ( n) − aδ ( n − 1)
Difference equation: y ( n) = x( n) − ax( n − 1)
Q.74 Determine the autocorrelation of the sequence {1, 1, 2, 3} . (8)

Ans:x(n) = (1, 1, 2, 3)

Since rxx (k ) = ∑ x ( m) x ( m − k ) = r
m = −∞
xx ( − k )

r (0) = x (0) x(0) + x (1) x(1) + x(2) x(2) + x(3) x(3) = 15


r (1) = x (0) x(1) + x (1) x (2) + x( 2) x(3) = 9 = r (−1)
r ( 2) = x(0) x (2) + x(1) x(3) = 5 = r ( −2)
r (3) = x(0) x (3) + x(1) x(3) = 3 = r (−3)
r (≥ 4) = 0
Thus
r (n) = [3, 5, 9, 15, 9, 5, 3]

Q.75 Determine the cross correlation of the processes
x1 (t ) = A cos(2πf c t + θ ) and x 2 (t ) = B sin( 2πf c t + θ ) ,
where θ is an independent random variable uniformly distributed over the
interval (0,2π ) . (8)

Ans:x1(t) = A cos (2πfct + θ) and x2(t) = B sin (2πfct + θ)


Rxx(τ) = E{A cos {2πfc(t + τ)+θ}B sin (2πfct + θ)}
AB
= E[sin {2πfc(-τ)} + sin {2πfc(2t + τ + 2θ)}]
2
π
AB AB 1
=-
2
sin(2πfcτ) +
2 2π −π ∫
sin{2πfc(2t +τ+2θ)}dθ

AB
=- sin (2πfcτ)
2

sin πt ∞
 n
Q.76 A signal x(t ) = is sampled by p (t ) = ∑ δ  t − . Determine and
πt n = −∞  2
sketch the sampled signal and its Fourier transform. (8)

sin πt ∞
 n
Ans: x(t ) = sampled by p (t ) = ∑ δ  t − . Thus, the sampled signal
πt n = −∞  2
is
n
sin π  
x (n) = 2 .
n
π 
2
DTFT of x( n) is a pulse

57
AE06/AC04/AT04 SIGNALS & SYSTEMS

 π
2, ω ≤
X (ω ) =  2
0, otherwise

Q.77 Determine the Fourier transforms of (8)


(i) x1[n ] = sin nω0 and (ii) x2 [n] = (sin nω0 )u[n ]

e jω0 n − e − jω0 n
Ans: (i) x1 [n ] = sin nω 0 =
2j
=
2j
e [
1 jω0 n
− e − jω0 n ]
1
X (ω ) = [δ (ω + ω 0 ) − δ (ω − ω 0 )]
2j
e jω 0 nu ( n) − e − jω 0 nu ( n)
(ii) x2 [n] = (sin nω0 )u ( n) =
2j
1  1 1 
X (ω ) =  jω0 − jω
− − jω0 − jω 
2 j 1 − e e 1− e e 
1 jω  1 1 
= e  jω jω0
− jω − jω0 
2j e − e e −e 
 sin ω 0 
= e jω  jω

j 2ω
1 + e − 2e cos ω 0 
1 sin ω 0 
=  
2  cos ω − cos ω 0 

s 4 + 3s 3 − 4 s 2 + 5s + 5
Q.78 Find the inverse Laplace transform of X (s ) = for all
s 2 + 3s − 4
possible ROCs. (8)

s 4 + 3s 3 − 4 s 2 + 5 s + 5
Ans: H(s) =
s 2 + 3s − 4
5s + 5 5s + 5 3 2
= s2 + 2 = s2 + = s2 + + .
s + 3s − 4 ( s + 4)( s − 1) s + 4 s −1
d 2δ (t )
h (t ) = + 3e − 4t u (t ) + 2e t u (t ), ROC σ > 1
dt 2
d 2δ (t )
h (t ) = + 3e − 4t u (t ) − 2e t u (−t ), ROC − 4 < σ <1
dt 2
d 2δ (t )
h (t ) = 2
− 3e − 4t u (−t ) − 2e t u ( −t ), ROC σ < −4
dt

Q.79 Using Laplace transform, find the forced and natural responses of the system
d 2 y (t ) dy (t ) dx(t )
described by 2
+5 + 6 y (t ) = + 6 x(t ) when the input is a unit
dt dt dt
step function and the initial conditions of the system are y (0 + ) = 1 and
y ' (0 + ) = 2. (8)

Ans:Taking the unilateral Laplace transform of both sides of the given eqn,
we obtain
58
AE06/AC04/AT04 SIGNALS & SYSTEMS

( s 2 + 5s + 6)Y ( s ) − sy (0 + ) − y ' (0 + ) − 5 y (0 + ) = ( s + 6) X ( s) − x(0 + )


Solving for Y(s)
( s + 6) X ( s) − x(0 + ) sy (0 + ) + y ' (0 + ) + 5 y (0 + )
Y ( s) = +
( s + 2)(s + 3) ( s + 2)( s + 3)
The first term is associated with the forced response of the system, yF(t). The
second term corresponds to the natural response, yN(t). Substituting for X(s)
= 1/s, x(0+) = 1, y (0 + ) = 1 , y ' (0 + ) = 2 , we obtain
 6   s+7 
Y ( s) =   +  
 s ( s + 2 )( s + 3 )   ( s + 2)( s + 3 ) 
1 3 2   5 4 
Y ( s) =  − + + − 
 s s + 2 s + 3   s + 2 s + 3
( )
y (t ) = 1 − 3e −2t + 2e −3t u (t ) +(5 e −2t -4 e −3t ) u (t )
Thus,
(
y F (t ) = 1 − 3e −2t + 2e −3t u (t ) ,) y N (t ) = (5 e −2t -4 e −3t ) u (t ) .
1 + z −1
A casual system is described by H ( z ) = . For what values
Q.80 1 − az −1 1 − bz −1 ( )( )
of a and b will the system be (i) unstable, (ii) non-causal? (8)

z ( z + 1)
Ans: H ( z ) =
( z − a )( z − b)
The poles are z = a, b.
(i) Causal: both a and b ≤ 1. Stable: both a and b < 1.
(ii) Unstable: both or either a or b ≥ 1. Non-causal: both or either a or b > 1.
Q.81 Determine the ROC of aX ( z ) + bY ( z ) , given that
z 0.25 z
X (z ) = , 0.5 < z < 1.5, Y ( z ) =, z > 0.5 .
(z − 0.5)(z − 1.5) (z − 0.25)(z − 0.5)
For what relationship between a and b the ROC will be the largest? (8)

z
Ans:X(z) = , 0.5<z< 1.5
( z − 0.5)( z − 1.5)
0.25 z
Y(z) = , z> 0.5
( z − 0.25)( z − 0.5)
 0.25a + 0.375b 
z (a + 0.25b) z − 
 a + 0.25b 
aX(z) + bY(z) =
( z − 0.5)( z − 1.5)( z − 0.25)
Since ROC is decided by the three poles, ROC of aX(z) + bY(z) is 0.5<z<
1.5. However, there is a possibility of cancellation of one of the poles by the
created zero. If the pole at 0.5 can be cancelled, then we shall have the ROC
given by 0.25 < |z| < 1.5 the maximum stretch. The condition is
0.25a + 0.375b
= 0.5 , i.e., a = b.
a + 0.25b

Q.82 Find whether the function y(t) = x(t).cos(100πt) represent a Linear, Causal,
time invariant system. (8)

59
AE06/AC04/AT04 SIGNALS & SYSTEMS

Ans: y (t ) = x(t ) cos100πt


If the inputs are x1 (t ) and x 2 (t ) , then the corresponding outputs are
y1 (t ) = x1 (t ) cos100πt
y 2 (t ) = x 2 (t ) cos100πt .

Now if the input x3 (t ) is a linear combination of x1 (t ) and x 2 (t ) , i.e.,


x3 (t ) = ax1 (t ) + bx 2 (t )
where a and b are arbitrary scalars. Then
y 3 (t ) = x3 (t ) cos100πt = [ax1 (t ) + bx 2 (t ) ]cos100πt
= ax1 (t ) cos100πt + bx 2 (t ) cos100πt
= ay1 (t ) + by 2 (t ) .
Thus, we conclude that the system is linear.
Since the response depends only on the present values, the system is
causal.
Since y (t − τ ) = x (t − τ ) cos 100π (t − τ ) ≠ y (t ) , the system is time varying.

Q.83 Find the even and odd parts of the following functions (4)
(i) f (t ) = t sin t (ii) f (t ) = a0 + a1t + a2t 2

Ans:
1
f o (t ) = [ f (t ) − f (−t )] f e (t ) = 1 [ f (t ) + f (−t )]
2 , 2
(i) f (t ) = t sin t
1
f o (t ) = [t sin t − (−t ) sin(−t )] = 0
Here 2
1
f e (t ) = [t sin t + (−t ) sin(−t )]
2 = t sin t

(ii) f (t ) = a + a1 (t ) + a 2 t 2

f o (t ) =
1
2
[
{a + a1 (t ) + a2 t 2 }− {a + a1 (−t ) + a2 (−t ) 2 } = a1t ]
Here
f e (t ) =
1
2
[
{a + a1 (t ) + a2 t 2 }+ {a + a1 (−t ) + a2 (−t ) 2 } = a0 + a 2 t 2 ]
Q.84 Find the average power of the signal x(t ) = (e −5t + 1) u (t ) . (4)

Ans:Average power over an infinite interval


T
1
P∞ = Lt T →∞ ∫ x (t )
2
dt
2T −T
T T

∫ (e )u(t )dt = 1 ∫ (e )
1 −10t
= Lt T →∞ −10t
+ 1 + 2e −5 t
+ 1 + 2e −5t dt
2T −T
2T 0
1
=
2 W

60
AE06/AC04/AT04 SIGNALS & SYSTEMS

Q.85 Find the Fourier Series of the following periodic wave form and hence draw
the spectrum. (8)

π
f (t ) = A cos t,
Ans:The function 4 − 2 < t < 2 has even symmetry.
T
1
Fn (t ) = ∫ f (t )e − jnω0t dt
T 0
π
2
1  π  − jn t
= ∫  A cos t e 2 dt
4 − 2 4 
2 j t 
π π
−j t
A e 4 +e 4  − jn π2 t
= ∫ e dt
4 − 2 2 
 
 π 2  − j π t (1− 2 n )  2 
 j 4 t (1− 2 n )   e 4  
A e
=   +  
8  π π
(
j 1 − 2n )   (
− j 1 − 2n   )
 4  − 2  4  − 2

 π π 
sin (1 − 2n) sin (1 + 2n) 
A 2 2
=  + 
π  (1 − 2n ) (1 + 2n ) 
 
 
2A 2A 8A
∴ F0 = F1 = F2 =
π , 3π , 15π

Q.86 Find the trigonometric Fourier series of the following wave form. (8)

Ans:Here T = 1, ω 0 = 2π , f (t ) = 2Vt .
Since the function exhibits an odd symmetry, a o = a n = 0 .
T /2
4
bn = ∫ f (t ) sin nω 0 tdt
T 0

61
AE06/AC04/AT04 SIGNALS & SYSTEMS

0.5
= 4 ∫ (2Vt ) sin 2πntdt
0

 t (− cos 2πnt ) 0.5 0.5 (− cos 2πnt ) 


= 8V  − ∫ 1. dt 
 2πn 0 0
2πn 
 t ( − cos 2πnt ) 0.5 ( − sin 2πnt ) 0.5

= 8V  −. 
 2πn 0 2πn.2πn 0 
2V
=− cos πn + 0
πn
Now

2V ∞
f (t ) = ∑ bn sin 2πnt = ∑ − cos πn. sin 2πnt
1 π 1

2V  1 1 
= sin 2πt − sin 4πt + sin 6πt + ....
π  2 3 

Q.87 Define signum and unit step functions? Find the Fourier transforms of
these functions. (8)

Ans:Signum function
1, t >0
Sgm (t ) = 
− 1, t < 0

F (ω ) = Lt a →0 ∫ e
−a t
Sgm(t )e − jωt dt
FT [Sgm] −∞

 0 ∞

= Lt a →0  ∫ − e −( a + jω )t dt + ∫ e − ( a + jω )t dt 
−∞ 0 
1 1 2
= + =
jω jω jω
2
∴ F (ω ) =
ω
Unit Step Function
1 t ≥ 0
u (t ) = 
0 t < 0
Since Sgn (t) = 2u (t ) − 1 ,
1
[sgn(t ) + 1]
u (t )
=
2
1 2  1
∴ FT of u (t ) =  + 2πδ (ω ) = + πδ (ω )
2  jω  jω

Q.88 Determine the Fourier transform a two-sided exponential function


x(t ) = e and draw its magnitude spectrum.
−t
(8)

−t
Ans: f (t ) = e

62
AE06/AC04/AT04 SIGNALS & SYSTEMS

∞ 0 ∞

∫e ∫e + ∫ e −(1+ jω )t dt
−t − jωt (1− jω ) t
F (ω ) = e dt =
−∞ −∞ 0

1 1 2
= + =
1 − jω 1 + jω 1 + ω 2
2
F (ω ) =
∴ 1+ ω2

Q.89 ind the Discrete Fourier transform of the following sequences.


(i) x(n ) = a n , 0 < a < 1 (Find N point DFT)
π
(ii) x(n ) = cos n (Find 4 point DFT) (8)
4

Ans:
N −1
X (k ) = ∑ x ( n) e
n =0
− j 2πnk / N
,
k = 0,1,..., N − 1
(i)
N −1
= ∑a e
n=0
n − j 2πnk / N
,

N −1
(
1 − ae − j 2πk / N ) N

= ∑ (ae
n=0
− j 2πnk / N n
) =
1 − ae − j 2πk / N
1− aN
= ,
1 − ae − j 2πk / N k = 0, 1, ...., N − 1
(ii) N = 4
π π 3π
x(n) = cos 0, cos , cos , cos
4 2 4
= 1, 0707, 0, -0,707
N −1

X (k ) =
∑ x ( n) e
n =0
− j 2πnk / N
,
k = 0,1,..., N − 1
3

∑ x ( n) e
n =0
− j 2πnk / N
,
k = 0, 1, 2, 3
=
∴ X (k ) = {X (0), X (1), X (2), X (3)}
= {1, 1 − j1.414, 1, 1 + j1.414}

Q.90 (i) Find the circular convolution of the following sequence (rectangular)
1, 0 ≤ n ≤ N − 1
x1 (n) = x 2 ( n) = 
0, otherwise
(ii) Compute the DFT of
a) x (n ) = δ (n )
b) x(n ) = δ (n − n 0 ) (4)

N −1
N , k = 0
X 1 ( k ) = X 2 (k ) = ∑W
n =0
N
nK
=
0 0therwise
Ans: (i)

63
AE06/AC04/AT04 SIGNALS & SYSTEMS

N 2 , k = 0

X 3 ( k ) = X 1 ( k ) X 2 (k ) = 0, elsewhere
x3 ( n ) = N , 0 ≤ n ≤ N − 1
(ii) a) X ( k ) = DFT [δ ( n)] = 1
− jωno
b) Since x[n − no ] ↔ e X ( jω ), we get

DFT [δ (n − n0 ] = e
− jωmo

Q.91 Find the Nyquist frequency of the following signals.

(i) Sa (100 t ) (ii) Sa 2 (100 t )


(iii) 25 cos(500 πt ) (iv) 10 sinc(2t ) (8)

sin x
x=
Ans: Sa x
(i) This function will have frequency response a rectangular pulse with
100 100
fm = , fs = 2 fm =
maximum frequency 2π π Hz
(ii) The sin2(2πft) = (1-cos 4πft)/2, the maximum frequency will be 2f.
100 200
fm = , fs = 2 fm =
π π Hz
500
fm = ,
(iii) 2π f s = 2 f m = 500 Hz
sin πt sin 2πt
sinc t = . 10
(iv) πt Thus 10 sinc(2t) = 2πt . Its frequency response will
be a rectangular pulse X ( f ) such that the maximum frequency f m = 1 Hz.
Hence sampling frequency f s = 2 f m = 2 Hz.

Q.92 Define ideal low pass filter and show that it is non-causal by finding its
impulse response. (8)

Ans:
e − j 2πft0 ,− B ≤ f ≤ B
H( f ) = 
 0, f > B
B
e j 2πf (t −t0 ) B
h(t ) = − B ∫ e j 2πf ( t − t 0 )
df =
−B
j 2πf (t − t 0 ) − B

=
1
j 2π (t − t 0 )
[ ]
e j 2πB (t −t0 ) − e − j 2πB (t −t0 )

sin 2πB (t − t 0 )
= 2B = 2 BSinc[2 B (t − t 0 )]
2πB (t − t 0 )
The system is non-causal as there is some response before t = 0.

Q.93 Obtain the Laplace transform of the square wave shown. (8)

64
AE06/AC04/AT04 SIGNALS & SYSTEMS

Ans:
2T 4T ( n + 2 )T

[ f (t )] = ∫ f (t )e − st
dt + ∫ f (t )e dt +.... +
− st
∫ f (t )e
− st
dt
£ 0 2T nT

2T
 2T  − 2 sT  2T 
= ∫ f (t )e dt + ∫ f (t )e dt .e
− st
 − st
 + ... +  ∫ f (t )e − st dt .e − nsT
0 0  0 
 2T

( )
= 1 + e − 2 sT + e − 4 sT + .... + e − nsT  ∫ f (t )e − st dt 
0 
1  2T
 1  T 2T

1 − e − 2 sT  ∫0 ∫ ∫
− st − st − st
=  f (t ) e dt  = − 2 sT  0 .5 e dt + 0 . 5e dt 
 = 1− e 0 T 
 
=
1

1 − e − 2 sT  s
0 .5
(
e − sT − 1 +
0 .5
) (
e − 2 sT − e − sT  )
s 

=
1
− 2 sT 
0 . 5
(1 − e −sT )  2

1− e  s 
0.5 1 − e  − sT
=  
s 1 + e − sT 

Q.94 Find the inverse Laplace transforms of the following functions.

s2  s +1 
(i) (ii) ln   (8)
(s + a ) 2
+b 2
 s+ 2
Ans:
−1 s2  s+a a b 
= s − 2 
(i) £ (s + a ) 2
+b 2
 (s + a) + b
2 2
b ( s + a) + b 
2

d  a 
f (t ) + f (0 + ) f (t ) = e − at cos bt − e − at sin bt  u (t )
= dt where  b 
  a 2 − at  
 (
− be − at
sin bt − ae − at
cos bt + 
 − ae − at
cos)bt + e sin bt  + δ (t )u (t )
=   b  
a 2

=  sin bt − 2a cos bt − b sin tbt e − at u (t ) + δ (t )
 b 
s +1
=
(ii) F ( s ) = ln s + 2 ln ( s + 1) − ln ( s + 2)

65
AE06/AC04/AT04 SIGNALS & SYSTEMS

dF 1 1
∴ = −
ds s + 1 s + 2
 dF 
(
∴   = e −t − e − 2t u (t ) )
 ds 
Hence,
 dF 
−1
[F (s)] = − 1  
−1

£ t £  ds 
1
[
− e −t − e − 2t u (t ) ]
= t

Q.95 Obtain the z transforms and hence the regions of convergence of the
following sequences.
(i) x(n ) = [u (n ) − u (n − 10 )] 2 − n (ii) x (n ) = cos (π n ) u (n ) (8)

Ans: (i) x (n) = [u (n) − u (n − 10)]2 − n


9
Z [x( n)] = ∑ 2 − n Z − n
n=0
n
 1 
1−  
−n =
 2z 
9
 1 
= ∑  1 −
1
z >
1
n =0  2 z  2 z , ROC is 2
e jπn + e − jπn
x(n) = cos(πn )u (n) = u ( n)
(ii) 2
1  9 jπn − n − jπn − n  1  ∞ jπ −1 ∞
n
∑( ) + ∑ (e )
9
∴ Z [x (n)] = ∑ ∑
n − jπ
 e z + e z  =  e z z −1 
2  n=0 n =0  2  n =0 n =0 
1 1 1 
=  jπ −1
+ − jπ −1 
2 1 − e z 1− e z 
1 1 1  z
=  + −1 
=
2 1 + z −1
1+ z  z +1, z >1
ROC is

Q.96 A second order discrete time system is characterized by the difference


equation y (n ) − 0.1 y (n-1)-0.02 y (n-2 ) = 2 x(n ) − x(n − 1) . Find y (n ) for n ≥ 0
when x(n) = u(n) and the initial conditions are given as y(–1) = –10, y(–2)
= 20 (8)

Ans:
Since y ( n) − 0.1 y ( n − 1) − 0.02( n − 2) = 2 x( n) − x( n − 1) ,
[ ] [
Y (z ) − 0.1 Y (−1) + z −1Y (z) − 0.02 Y (−2) + z −1Y(−1) + z −2 Y (z) ]
−1
= 2X ( z ) − z X ( z )
Substituting the initial values and rearranging, we get
[ ]
Y ( z ) 1 − 0.1z −1 − 0.02 z − 2 + 1 − 0.4 + 0.2 z −1 =
2z

1
z −1 z −1

66
AE06/AC04/AT04 SIGNALS & SYSTEMS

( z − 0.2)( z − 0.1) 2 z − 1 0.2


Y ( z) = = − − 0.6
z2 z −1 z
(2 z − 1) z 2 0.2 0.6
Y ( z) = − −
( z − 1)( z − 0.2)( z − 0.1) z ( z − 0.2)( z − 0.1) ( z − 0.2)( z − 0.1)
= Y1 ( z ) + Y2 ( z ) + Y3 ( z )
Now
Y1 ( z ) ( 2 z − 1) z
=
z ( z − 1)( z − 0.2)( z + 0.1)
By partial fraction expansion, we get
1.13 z 0.5 z 0.36 z
Y1 ( z ) = + +
( z − 1) ( z − 0.2) ( z + 0.1)
Similarly
2 z 2 z
Y2 ( z ) = +−
3 ( z − 0.2) 3 ( z + 0.1)
z z
Y3 ( z ) = 0.4 + 0.2
( z − 0.2) ( z + 0.1)
Thus,
1.13 z 0.56 z 0.83 z
Y (z) = − +
( z − 1) ( z − 0.2) ( z + 0.1)
Hence
[
y (n) = 1.13 − 0.56(0.2) + 0.83(0.1) u ( n)
n n
]
Q.97 A continuous random variable has a pdf f (x ) = Kx 2e− x ; x ≥ 0 . Find K, and
mean and variance of the random variable. (8)

Ans:
By the property of PDF

∫ Kx
2 −x
e dx = 1
0

Or, 2K = 1 → K = ½
Mean value of x is


∫ xf ( x)dx = ∫ xKx e dx = 0.5∫ x e dx = 3
2 −x 3 −x
E ( x) =
0
Rx 0

Now
∞ ∞
E(x 2 ) = ∫x f ( x )dx = ∫ x 2 Kx 2 e − x dx =0.5 ∫ x 4 e − x dx = 12
2
0 0
Rx

Variance of x is
V ( x) = E ( x 2 ) − {E ( x)} = 12 - 9 = 3.
2

Q.98 Find the autocorrelation of the following functions:


(i) g (t ) = e − at u (t )
t  t 
(ii) g(t) = A Π   where A Π   is a rectangular pulse with period T and
T T

67
AE06/AC04/AT04 SIGNALS & SYSTEMS

magnitude A. (8)

Ans:
(i) g (t ) = e − at u (t )

R g (τ ) = ∫ −∞
g (t ) g (t − τ )dt
∞ ∞ e − aτ

−∞
e − at e − a ( t −τ )u (t )dt = e aτ ∫0
e − 2 at dt =
2a
t 

(ii) g (t ) = A Π T

R g (τ ) = ∫
−∞
g (t ) g (t − τ )dt
=
R g (−τ )

For τ < T
T

= ∫τ
R g (τ ) A 2 dt = A 2 (T − τ )

For τ ≥ T

=∫
R g (τ ) g (t ) g (t − τ )dt 0, τ ≥T
T =
 A 2 (T − τ ), τ < T
=
0 τ ≥ T.

Q.99 Find the Fourier series of the following periodic impulse train. (8)

Ans:
1 T0 / 2 I
A0 =
T0 ∫ −T0 / 2
δ (t ) dt =
T0
2I T0 / 2 2πnt 2I
An =
T0 ∫−T0 / 2 δ (t ) cos T0 dt = T0
2I T0 / 2 2πnt
Bn =
T0 ∫−T0 / 2 δ (t ) sin T0 dt =0
2πnt
I 2I ∞
2πnt I ∞
∴ x(t ) = +
T0 T0

n =1
cos
T0
=
T0
∑e
n = −∞
T0

Q.100 The Magnitude and phase of the Fourier Transform of a signal x(t) are
shown in the following figure. Find the signal x(t).

Ans:

68
AE06/AC04/AT04 SIGNALS & SYSTEMS

X ( jω ) = π , − W ≤ ω ≤ W
π / 2, ω<0
and φ ( jω ) = ∠ X ( jω ) = 
− π / 2, ω > 0
πe jπ / 2 , − W ≤ ω ≤ 0
Thus X ( jω ) =  − jπ / 2
πe , 0 ≤ω ≤W
1 ∞
x(t ) = F −1 [X ( jω )] = ∫ X ( jω )e jωt dω
2π −∞

1  0 jπ / 2 jωt
πe e dω + ∫ πe − jπ / 2 e jωt dω 
W
= ∫
2π  −W 0 
1
=  ∫ e j (π / 2−ωt ) dω + ∫ e − j (π / 2− dω 
W W jωt )

2 
 0 0 
1 W
 e j (π / 2−ωt ) + e − j (π / 2− jωt ) dω  = Wcos(π / 2 − ωt )dω = W(sin ωt )dω
=
2  ∫
0    0 ∫ 0 ∫
 Wt  W t  Wt 
2
1 1
= [1 − cos Wt ] = 2 sin 2   = 2 sinc 2  
t t  2  2π  2π 

Q.101 Find the Discrete Time Fourier Transforms of the following signals and draw
its spectra. (8)
(i) x1 (n ) = a
n
a <1

(ii) x 2 (n ) = cos ω 0 n where ω 0 = .
5
Ans:
n
(i) x(n) = a , a < 1
∞ −1 ∞ ∞ ∞
( ) ∑a
X e jω =
n
e j ωn = ∑a −n
e jωn + ∑a n
e j ωn = ∑ (ae ω ) + ∑ (ae ω )
−j n j n

n = −∞ n = −∞ n=0 n =1 n=0

ae − jω 1 1 − a2
= + =
1 − ae − jω 1 − ae jω 1 − 2a cos ω + a 2
1
[
(ii) x(n) = cos ω 0 n = e jω0 n + e − jω0 n
2
]
∞ ∞
 2π   2π 
X (e j ω ) =
l = −∞ 

πδ ω −
5
− 2πl  + πδ ω +
 l = −∞  5
− 2πl 


 2π   2π 
= πδ ω −  + πδ ω + , −π ≤ ω < π
 5   5 

Q.102 The frequency response for a causal and stable continuous time LTI system
1 − jω
is expressed as H ( jω ) = . (8)
1 + jω
(i) Determine the magnitude of H ( jω )
(ii) Find phase response of H ( jω )
(iii) Find Group delay.

Ans:

69
AE06/AC04/AT04 SIGNALS & SYSTEMS

(i) [H (ω )] = 1 − jω =
1+ ω2
=1
1 + jω 1+ ω2
(ii) ∠ H ( jω ) = tan −1 (−ω ) − tan −1 (ω ) = −2 tan −1 (ω )

(iii) Group delay = −


d

H ( jω ) = −

d
(− 2 tan −1 ω ) =
2
1+ ω2
Q.103 Find the Nyquist rate and Nyquist interval for the continuous-time signal
1
x (t ) = cos(4000πt ) ⋅ cos(1000πt ) . (4)

Ans:
1 1
x(t ) = cos 4000πt. cos1000πt = [cos 5000πt + cos 3000πt ]
2π 4π
The highest frequency present is ωh = 5000π , i.e., fh = 2.5 kHz. Nyquist rate is
5 kHz and Nyquist interval = 1/5 k = 0.2 msec.

Q.104 Consider a discrete-time LTI system with impulse response h (n ) given by


h(n ) = α n u (n ) .
Determine whether the system is causal, and the condition for stability. (4)

Ans:
Since h(n) = 0 for n < 0, the system is causal.
Now
∞ ∞ ∞
1
∑ h( k ) = ∑ α k u ( n) = ∑ α
k
= , α < 1.
k = −∞ k = −∞ k =0 1− α
Thus the system is stable if α < 1 .

Q.105 Check for Causality, Linearity of the following signals. (8)


(i) y (t ) = x t( ) (ii) y (t ) = x(t 2 )
n
(iii) y (t ) = 10 x(t + 2 ) + 5 (iv) y[n] = ∑ x[n]
k = −∞
Ans:
(i) Non-causal: y (0.01) = x(0.1), i.e., y depends upon the future value of x.
linear: it is of the form y = mx.
(ii) Non-causal: y ( 2) = x(4), i.e., y depends upon the future value of x. linear:
it is of the form y = mx.
(iii) Non-causal: y(t) depends upon x(t+2) the future value of x(t). Non-linear:
it is of the form y = mx + c.
(iv) Non-causal: It has the value for n < 0. Linear : it is of the form y = mx.

Q.106 Determine the Laplace transform of the following functions. (6)


(i) x(t ) = cos 3 (3t ) (ii) x(t ) = t sin at
Ans:
(i)
1
x(t ) = cos 3 t = [cos 9t + 3 cos 3t ]
4
70
AE06/AC04/AT04 SIGNALS & SYSTEMS

1 s 3s  1  s 3s 
∴ x(t ) =  + 2 =  2 + 2 .
4 s + 9
2 2
s + 3  4  s + 81 s + 9 
2 

(ii) x(t ) = t sin at


d d  a  2as
∴ £ x(t ) = − £ (sin at ) = −  2 2 
= 2 .
ds ds  s + a  ( s + a 2 ) 2

2 1
Q.107 The transfer function of the system is given by H (s ) = +
s+3 s−2
Determine the impulse responses if the system is (i) stable (ii) causal. State
whether the system will be stable and causal simultaneously. (10)

Ans:There are following three possible impulse responses.


h1 (t ) = 2e −3t u (t ) + e 2t u (t ), σ > 2
h2 (t ) = 2e −3t u (t ) − e 2t u ( −t ), - 3 < σ < 2
h3 (t ) = −2e −3t u ( −t ) − e 2t u (−t ), σ < −3
h1(t) is causal but unstable due to the pole at s = 2, h2(t) is non-causal due to
the pole at s = 2 but stable, and h3(t) is also non-causal due to both the poles
but stable. Thus, the system cannot be both causal and stable simultaneously.
Q.108 Determine the inverse Z transform of the following X(z) by the partial
fraction expansion method. (8)
z+2
X ( z) =
2z 2 − 7 z + 3
1 1
if the ROCs are (i) z > 3 , (ii) z < , (iii) < z < 3
2 2

z+2
Ans: X ( z ) =
2z − 7z + 3
2

X ( z) z+2 z+2 2/3 1 1/ 3


= = = − +
z (
2
)
2 z z − 7 z + 3 2 z (z − 0.5)( z − 3) z z − 0.5 z − 3
2 z (1 / 3) z
Or X ( z ) = − + , poles are p1 = 0.5, p 2 = 3
3 z − 0.5 z − 3
(i) When z > 3 all poles are interior, x(n) is causal.
n
2 1 1
Therefore, x(n) = δ (n) −   u (n) + (3) n u (n)
3 2 3
1
(ii) When z < , all poles are exterior, x(n) is non-causal.
2
n
2 1 1
∴ x(n) = δ (n) +   u (−n − 1) − (3) n u (−n − 1)
3 2 3
1
(iii) When < z < 3, p1 is interior and p 2 is exterior.
2
n
2 1 1
∴ x(n) = δ (n) −   u (n) − (3) n u (− n − 1)
3 2 3

Q.109 A Causal discrete-time LTI system is described by


71
AE06/AC04/AT04 SIGNALS & SYSTEMS

3 1
y(n ) − y (n − 1) + y (n − 2 ) = x(n )
4 8
where x(n ) and y (n ) are the input and output of the system, respectively.
(i) Determine the H(z) for causal system function
(ii) Find the impulse response h(n) of the system
(iii) Find the step response of the system (8)

Ans:
3 −1 1
i) Y (z) − z Y ( z ) + z − 2Y ( z ) = X ( z )
4 8
Y ( z) 1 z2 1
∴ H ( z) = = = ,z > .
1 − z −1 + z − 2  z −  z − 
X ( z) 3 1 1 1 2
4 8  2  4
H ( z) z 2 1
(ii) = = −
z  1  1 1 1
 z −  z −  z − z−
 2  4 2 4
  1 n  1 n 
∴ h(n) = 2  −    u (n)
  2   4  
z
(iii) Here X ( z) = , z >1
z −1
z3
Y ( z) = X ( z)H ( z) = , z >1
1 1
( z − 1)( z − )( z − )
2 4
8 z z 1 z
= −2 + , z >1
3 z −1 1 3 1
z− z−
2 4
8  1  n
11 
n

y (n) =  − 2  +    u (n)
 3  2  3  4  

Q.110 A random variable X has the uniform distribution given by


1
 , for 0 ≤ x ≤ 2π
f X ( x ) =  2π
0, otherwise
Determine mean, mean square, Variance. (10)
∞ 2π
1
Ans: Mean m x =
−∞
∫ xf x ( x)dx = ∫ x 2π dx = π
0
∞ 2π
1 4
Mean square X 2 == E ( X 2 ) = ∫ x f x ( x)dx = ∫x dx = π 2
2 2

−∞
2π 0
3
4 1
Variance: σ x 2 = E( X 2 ) − mx 2 = π 2 −π 2 = π 2
3 3

72
AE06/AC04/AT04 SIGNALS & SYSTEMS

π
σx =
3
Q.111 Discuss the properties of Gaussian PDF. (6)

Ans:
Property 1: The peak value occurs at x = m, i.e., mean value
1
f x ( x) = at x = m (mean)
σ 2π
Property 2: Plot of Gaussian PDF exhibits even symmetry around mean value,
i.e.,
f x (m − σ ) = f x ( m + σ )
Property 3: The mean under PDF is 1 / 2 for all values of x below mean
value and ½ for all values of above mean value, i.e.,
1
P ( X ≤ m) = P ( X > m) =
2
Q.112 A stationary random variable x(t) has the following autocorrelation
function
R x (τ ) = σ 2 e where σ 2 , µ are constants. Rx (t ) is passed through a
−µ τ

filter whose impulse response is h(τ ) = αe −ατ u (τ ) where α is a


constant, u (τ ) is unit step function.
(i) Find power spectral density of random signal x(t).
(ii) Find power spectral density of output signal y(t). (8)


Ans: (i) S X (ω ) = FT[RX (t )] = ∫ R X (τ )e − jωτ dτ
−∞
∞ 2 µσ 2
= ∫
−∞
σ 2e µ τ e − jωτ dτ =
µ2 +ω2
∞ α
(ii) H (ω ) = FT[h(τ )] = ∫ αe −ατ u (τ )e − jωτ dτ =
−∞ α + jω
2
2 α α 2 µσ 2
S y (ω ) = H (ω ) S X (ω ) = S X (ω ) = 2
α + jω α +ω2 µ2 +ω2

Q.113 Determine the convolution of the following two continuous time functions.
x(t ) = e − at u (t ) , a > 0 and h(t ) = u (t ) (8)
Ans:
∞ ∞ t
∫ ∫ e −aλ u (λ )u (t − λ )dλ = ∫e
− aλ
y (t ) = h(t ) * u (t ) = x (λ ) h(t − λ ) dλ = dλ
−∞ −∞ o

=−
a
e [ ]
1 −aλ 1 1
[
= 1 − e −at .
0 a
]

Q.114 Determine signal energy and power of the following signals


(i) x(n ) = u (n ) (ii) x(t ) = e −3t (8)
Ans:

73
AE06/AC04/AT04 SIGNALS & SYSTEMS

∞ ∞
(i) E= ∑
n = −∞
x 2 [n] = ∑ u [n] = 1
n = −∞
2

N N
1 1
P = lt N →∞
2N

n=− N
2
x [n ] = lt N →∞
2N
∑ u [n] = 1.
n=− N
2

T T

∫ [e ] dt = ∫ [e ]dt = − 6 [e ]
− 3t 2 1 −6 t − 6T
(ii) E= − e 6T = ∞
−T −T

P = LtT → ∞
1
2T
E = LtT → ∞ −
1 − 6T
12T
e − e 6T = ∞ [ ]

Q.115 Find the DTFT of the sequence x(n) = u(n). (4)

Ans:
∞ ∞
DTFT x(n) = X (e jω ) =
1
∑ x(n)e − jωn = ∑1.e − jωn =
n = −∞ n =0 1 − e − jω
It is not convergent for ω = 0.
e jω / 2
∴ X e jω =( ) ω
, ω ≠ 0.
2 sin
2
Find the inverse Fourier transform of δ (ω ) . (4)
Q.116
Ans:

F [δ (ω )] =
−1 1

2π − ∞
δ (ω )e jωt dω =
1 j ωt

e [ ] ω =0 =
1

Q.117 Check whether the following signals are energy or power signal and hence find the
corresponding energy or power. (6)
(i) x(t ) = Ae −α (t )
⋅ u (t ), α > 0
(ii) x(t ) = cos ω 0 t
2

Ans:
(i) x(t ) = Ae −αt u (t ), α > 0

e −2αt ∞ A
2


2
Then E= x(t ) dt = A2 =
−∞
− 2α 0 2α
Since 0 < E < ∞, x(t ) is an energy signal.

(ii) Since x(t ) = cos 2 ω o t is a periodic function, it is a power signal.


To / 2
1
P = Lt To → ∞
To ∫
− To / 2
x 2 (t )dt

To / 2 To / 2

∫ [ ] ∫ [cos ]
1 2 1
= Lt T → ∞ cos ωot dt = Lt T → ∞
2 4
ωot dt
To − To /2
To − To / 2
To / 2
1 1 3
= Lt T → ∞
To ∫ 8 [3 + 4 cos 2ω t + cos ω t ]dt = 8
− To / 2
o o

74
AE06/AC04/AT04 SIGNALS & SYSTEMS

Q.118 Find the convolution of two rectangular pulse signals shown below. (10)

Ans:
For − ∞ ≤ t ≤ 4 and t ≥ 10 the output is 0.
t
For 4 ≤ t ≤ 6 , y (t ) = ∫ 2dt = 2(t − 4)
4
8
For 6 < t ≤ 8 , y (t ) = ∫ 2dt = 2(8 − 6) = 4
6
10
For 8 < t ≤ 10 , y (t ) = ∫ 2dt = 2(10 − t )
t

Thus y(t) is as shown in the figure.


2
Q 119 Given the Gaussian pulse x(t ) = e −π t , determine its Fourier transform. (8)

Ans:
∞ ∞ ∞
∫ ∫ ∫
2 2
X (ω ) = x(t )e − jωt dt = e −πt e − jωt dt = e − (πt + j ωt )
dt
−∞ −∞ −∞

2

 jω  ω2
Expressing πt 2 + jωt =  π t +  + ,

 2 π 
 4π
we have
 2  2
 jω  ω2 ω2  jω 
− πt + 
− − −  πt + 
∞ 
2 π 
e 4π dt = e 4π
∞ 
2 π 
X (ω ) = ∫
−∞
e  

−∞
e  
dt

Let u = πt + ,
2 π
ω2 ω2
− −
X (ω ) = e 4π
2 ∞
− (u )2 du = e 4π
2 π − πf 2
Then
π ∫
0
e
π 2
=e

Q.120 Find the exponential Fourier series of the following signal. (8)

75
AE06/AC04/AT04 SIGNALS & SYSTEMS

Ans:
Here T = 1sec, ω o = 2π ,
10
x(t ) = t = 10t
T
T 1
1
Cn = ∫
T 0
x(t )e − jnωot dt = ∫ 10te − jn 2πt dt
0

 te − j 2πnt 1 1 e − j 2πnt 
= 10  −∫ dt 
 − j 2πn 0 0 − j 2πn 
 e − j 2πnt e − j 2πnt 
1

= 10  + 
 − j 2πn 4π n 0 
2 2

5
= j
πn
∞ ∞
5
Now x(t ) = ∑ C n e − j 2πnt = ∑ j e j 2πnt ;
n = −∞ n = −∞ πn

π
 n≥0
θ n = tan −1 ∞ =  2
π
− n≤0
 2

Q.121 State and prove the following properties of DTFT. (6)


(i) Time shifting, frequency shifting
(ii) Conjugate symmetry
(iii) Time reversal.

Ans:
x ( n ) ↔ X (e jω )
ωno
(i) Time shfting: x ( n − no ) ↔ e − j X (e jω )
Frequency shifting: e jωo n x(n) ↔ X (e j (ω −ωo ) )
(ii) x * (n) ↔ X * (e − jω )
X (e jω ) = X * (e − jω ) , x (n) real
Even [x( n)] ↔ Re X (e jω ) [ ]
Odd [x(n)] ↔ j Im X (e jω ) [ ]
(iii) x ( − n ) ↔ X (e − jω )

Q.122 Consider a stable causal LTI system whose input x(n ) and output y (n ) are related
through second order difference equation
3 1
y (n ) −   y (n − 1) + y (n − 2 ) = 2 x(n ) .
4 8
n
1
Determine the response for the given input x(n ) =   u (n ) (10)
4
Ans:

76
AE06/AC04/AT04 SIGNALS & SYSTEMS

n
3 1 1
y (n) − y (n − 1) + y (n − 2) = 2 x(n) = 2  u (n)
4 8 4
Taking DTFT on both sides
3 1 2
Y (e jω ) − e − jω Y (e jω ) + e − 2 jω Y (e jω ) =
4 8 1 − jω
1− e
4
jω 2
Y (e ) =
 3 − j ω 1 − 2 j ω   1 − jω 
1 − 4 e + 8 e  1 − 4 e 
2
=
 1 − j ω   1 − jω   1 − jω 
 21 − e  1 − 4 e  1 − 4 e 
2
= 2
 1 − jω   1 − jω 
1 − 2 e  1 − 4 e 
8 4 2
= − −
 1 − j ω   1 − j ω   1 − jω  2
1
 2 − e   41 − e  1 − e 
 4 
Taking inverse DFT
n n n
1 1 1
y (n) = 8  u (n) − 4  u (n) − 2(n + 1)  u (n)
2 4 4

Q.123 A continuous time signal is x(t ) = 8 cos 200πt (8)


(i) Determine the minimum sampling rate.
(ii) If fs = 400 Hz, what is discrete time signal obtained after sampling?
(iii) If fs = 150 Hz, what is discrete time signal obtained after sampling?

Ans:Here ω = 200π → f = 100 Hz


(i) Minimum sampling rate = 2 f = 2 × 100 = 200 Hz
f 100 1
(ii) = =
f s 400 4
1 πn
∴ x(n) = 8 cos 2πfn = 8 cos 2π n = 8 cos
4 2
f 100 2
(iii) Here = =
f s 150 3
2 4πn  2π  4πn
∴ x(n) = 8 cos 2πfn = 8 cos 2π n = 8 cos = 8 cos 2π − n = 8 cos
3 3  3  3

Q.124 State and prove Parseval’s theorem for continuous time periodic signal. (8)

77
AE06/AC04/AT04 SIGNALS & SYSTEMS

Ans:
Parseval’s theorem: The Parseval’s theorem states that the energy in the time-domain
representation of a signal is equal to the energy in the frequency domain representation
normalized by 2π.
Proof:
The energy in a continuous time non-periodic signal is
∞ 2
E = ∫ x(t ) dt.
−∞
2
Since x(t ) = x(t ) x * (t ) , from the Fourier series we get
1 ∞
x * (t ) =
2π − ∞ ∫
X * ( jω )e − jωt dω .
∞  1 ∞ 
Hence, E= ∫
−∞
x (t ) 
 2π − ∞ ∫
X * ( jω )e − jωt dω  dt

Now interchanging the order of the integrations, we get
X * ( jω )  x (t )e − jωt dt  dω =
1 ∞ ∞ 1 ∞
E=
2π ∫
−∞  ∫−∞  2π ∫−∞
X * ( jω ) X ( j ω ) d ω

1 ∞

2
= X ( j ω ) dω .
2π − ∞

Thus, the energy in the time-domain representation of the signal is equal to the energy
in the frequency-domain representation normalized by 2π.
Q.125 Compute the magnitude and phase of the frequency response of the first order discrete
time LTI system given by equation (10)
y (n ) − Ay (n − 1) = Bx(n ) where A < 1 .

Ans: y ( n) = Ay ( n − 1) + Bx( n )
h(n) = BA n u (n)

B
H (e jω ) = ∑ h ( n) e
n = −∞
− j ωn

1 − Ae − e
= jω

− jωn
Since 1 − Ae = (1 − A cos ω ) + jA sin ω
1 − Ae − jωn = (1 − A cos ω ) + ( A sin ω ) = 1 + A
2 2 2
− 2 A cos ω

Angle (1 − Ae ) = tan 1 −AAsincosωω


− jωn −1

B
∴ H ( e jω ) =
1 + A − 2 A cos ω
2

and
angle H (e jω ) = Angle  B − tan −1 A sin ω 
1 − A cos ω 
.

Q.126 Determine the Fourier transform of unit step x(t ) = u (t ). (6)

Ans:

Fourier transform of x(t) is X (ω ) = ∫ −∞
x(t )e − jωt dt
Thus Fourier transform of u(t) is

78
AE06/AC04/AT04 SIGNALS & SYSTEMS

∞ ∞ 1
X (ω ) = ∫
−∞
u (t )e − jωt dt = ∫ 0
e − jωt dt =

.

Q.127 By using convolution theorem determine the inverse Laplace transform of the
following functions. (8)
1 1
(i)
s2 s2 − a2 ( ) (ii)
s 2 (s + 1)
Ans:
1
(i) F (s) = 2 2 = F1 ( s ) F2 ( s )
s (s − a 2 )
1 1
where F1 ( s ) = and F2 ( s) =
s2 s − a2
2

1
Thus f1 (t ) = t and f 2 (t ) = sinh (at )
a
t
Now £ −1 F ( s ) = ∫ f 1 (t − τ ) f 2 (τ )dτ
0
t
1
= (t − τ ) sinh (aτ )dτ

0
a
1
[sinh at − 1]
=
a3
1
(ii) F (s) = 2 = F1 ( s) F2 ( s)
s ( s + 1)
1 1
where F1 ( s ) = 2 and F2 ( s ) =
s ( s + 1)
Thus f 1 (t ) = t and f 2 (t ) = e − t
t
Now £ F ( s ) = ∫ f1 (t − τ ) f 2 (τ )dτ
−1

0
t t t
= ∫ (t − τ )e −τ dτ = ∫ te −τ dτ − ∫ τe −τ dτ = (−te −t + t ) + (te −t + e −t − 1) = t + e −t − 1
0 0 0

Q.128 Check the stability & causality of a continuous LTI system described as
(s − 2)
H (s ) = (8)
(s + 2)(s − 3)
Ans:
s−2 1 4 1 
Given H (s) = =  + .
( s + 2)( s − 3) 5  s + 2 s − 3 
The system has poles at s = -2, s = 3.
Thus, the response of the system will be
4 − 2t 1
h1 (t ) = e u (t ) + e3t u (t ), σ > 3
5 5
4 − 2t 1
h2 (t ) = e u (t ) − e3t u (−t ), - 2 < σ < 3
5 5

79
AE06/AC04/AT04 SIGNALS & SYSTEMS

4 1
h3 (t ) = − e − 2t u (−t ) − e 3t u ( −t ), σ < −2
5 5
Note that the response h1(t) is unstable and causal, h2(t) is stable and non-causal, h3(t)
is stable and non-causal. Thus, the system cannot be both stable and causal
simultaneously.

Q.129 Find the z -Transform X ( z ) and sketch the pole-zero with the ROC for each of the
following sequences. (8)
n n
1 1
(i) x(n ) =   u (n ) +   u (n )
2  3
n n
1 1
(ii) x(n ) =   u (n ) +   u (− n − 1)
 3 2
Ans:
n n
1 1
(i) x(n) =   u (n) +   u (n)
2  3
n
1 z 1
  u ( n) ↔ , Z >
2 z−
1 2
2
n
1 z 1
  u (n) ↔ , z >
 3 z−
1 3
3
 5
2 z z − 
Thus, X (z) =
z
+
z
=  12 
, Z >
1
1 1  1  1 2
z− z−  z −  z − 
2 3  2  3
n n
1 1
(ii) x(n) =   u (n) +   u (− n − 1)
 3 2
n
1 z 1
  u (n) ↔ , z >
 3 z−
1 3
3
n
1 z 1
  u (−n − 1) ↔ − , z <
2 z−
1 2
2
z z 1 z 1 1
∴ X (z) = − =− , < z <
1 1 6 1  1 3 2
z− z−  z −  z − 
3 2  2  3

z
Q.130 Determine the inverse z transform of x(z ) = if the regions of
2
3z − 4 z + 1
1 1
convergence re (i) z > 1, (ii) z < , (iii) < z < 1, (8)
3 3

80
AE06/AC04/AT04 SIGNALS & SYSTEMS

Ans:
 
X (z ) 1 1 z z 
F ( z) = = 2 =  − 
z 3z − 4 z + 1 2  z − 1 z − 1 
 3
(i) ROC is z > 1
1 n  1   1 n 
n
1
∴ x ( n) = (1) u (n) −   u (n) = 1 −    u (n)
2  3  2   3  
1
(ii) ROC is z <
3
1 
n
1
∴ x(n) = − (1) +  
n
u (− n − 1)
2   3 
1
(iii) ROC is < z <1
3
1  1 
n
1
n
1
∴ x(n) =  − (1)n
u ( − n − 1) −   u ( n )  = − u ( − n − 1) +   u ( n) 
2  3  2  3  

−b x
Q.131 Consider the probability density function f X ( x ) = ae , where X is a random
variable whose allowable value range from x = −∞ to x = +∞ . Find
(i) Cumulative distribution function FX ( x ).
(ii) Relationship between a and b.
(iii) P[1 ≤ X ≤ 2] [assume b = 6] (8)
Determine mean, mean square and Variance.

Ans:
−b x
(i) f x ( x ) = ae
 f ( x ) = ae bx , − ∞ < x < 0
f x ( x) =  1 −bx
 f 2 ( x ) = ae , 0 < x < ∞
x a

∴ Fx1 ( x) = ae bx dx = e bx , x < 0
−∞ b


x a
Fx 2 ( x ) = ae − bx dx = 1 − ebx , x > 0
0 b
( )
 bx
a
 e , x<0
Thus, Fx ( x ) =  b
a
(
 1 − e bx , x > 0
b
)
0 ∞
(ii) Now ∫−∞
aebx dx + ∫0
ae − bx dx = 1
a a
→ + = 1 → 2a = b = 6 (given) → a = 3
b b
(iii) P[1 ≤ X ≤ 2] = ∫ 1
2
[
3e 6 x dx = 0.5e 6 e 6 − 1 . ]
 3e 6 x x<0
Now f x ( x) =  − 6 x
3e x>0
81
AE06/AC04/AT04 SIGNALS & SYSTEMS

0 ∞
Mean = E ( x) = ∫−∞
x3e 6 x dx + ∫
0
x3e − 6 x dx = 0

Variance of X = σ 2 = E (x 2 ) − [E (x )]2 = 3 ∫ x 2e6 x dx = ∫ x 2e − 6 x dx  − 0 =


0 ∞ 1
 −∞ 0  18

Q.132 Find the power spectral density for the cosine signal x(t ) = 8 cos[2π (3) t + π 3] and
also compute power in the signal. (8)

Ans:
Autocorrelation function of x(t )
T /2
1
T −T∫/ 2
R(λ ) = Lt T →∞ x(t ) x (t +λ )dt

π /2
1
8 cos(6πt + π / 3)8 cos[6π (t + λ ) + π / 3]dt
T −π∫/ 2
= Lt T →∞

T /2
32
[cos(12πt + 6πλ + 2π / 3) + cos 6πλ ]dt
T −T∫/ 2
= Lt T →∞

= 0 + 32 cos 6πλ = 32 cos 6πλ


PSD = F [R(λ )] = F [32 cos 6πλ ] = 32π [δ (ω − 2π ) + δ (ω + 6π )]
Power in the signal is R (0) = 32 cos 6πλ λ = 0 = 32 W

82

You might also like